Med Surg 2 - Test 1 Burns, Med Surg : Chapter 25 Burns, Med Surg burns test #1, Med Surg-Burn Quiz, Lewis: MED-SURG: Chapter 25: Burns, Med Surg - Burns CH 28

Ace your homework & exams now with Quizwiz!

nursing management for PT and OT

-ROM -splints

drug therapy (2)

-administer tetanus vaccine -administer pain med by IV

location- face and respiratory

-airway intubation should occur prior to swelling to prevent need for trach -HOB should be elevated above 30 degrees

A triage nurse in the ED admits a 50 year old male client with second degree burns on the anterior and posterior portions of both legs. Based on the Rule of Nines, what percentage of his body is burned? Record your answer using a whole number.

36 The anterior and posterior portions of one leg are 18%, if both legs are burned, the total is 36%.

In which order will the nurse take these actions when doing a dressing change for a partial-thickness burn wound on a patient's chest? (Put a comma and a space between each answer choice [A, B, C, D, E].) a. Apply sterile gauze dressing. b. Document wound appearance. c. Apply silver sulfadiazine cream. d. Administer IV fentanyl (Sublimaze). e. Clean wound with saline-soaked gauze.

ANS: D, E, C, A, B Because partial-thickness burns are very painful, the nurse's first action should be to administer pain medications. The wound will then be cleaned, antibacterial cream applied, and covered with a new sterile dressing. The last action should be to document the appearance of the wound.

Which patient should the nurse assess first? a. A patient with smoke inhalation who has wheezes and altered mental status b. A patient with full-thickness leg burns who has a dressing change scheduled c. A patient with abdominal burns who is complaining of level 8 (0 to 10 scale) pain d. A patient with 40% total body surface area (TBSA) burns who is receiving IV fluids at 500 mL/hour

ANS: A This patient has evidence of lower airway injury and hypoxemia and should be assessed immediately to determine the need for oxygen or intubation. The other patients should also be assessed as rapidly as possible, but they do not have evidence of life-threatening complications.

Which nursing action is a priority for a patient who has suffered a burn injury while working on an electrical power line? a. Obtain the blood pressure. b. Stabilize the cervical spine. c. Assess for the contact points. d. Check alertness and orientation.

ANS: B Cervical spine injuries are commonly associated with electrical burns. Therefore stabilization of the cervical spine takes precedence after airway management. The other actions are also included in the emergent care after electrical burns, but the most important action is to avoid spinal cord injury.

A young adult patient who is in the rehabilitation phase 6 months after a severe face and neck burn tells the nurse, "I'm sorry that I'm still alive. My life will never be normal again." Which response by the nurse is best? a. "Most people recover after a burn and feel satisfied with their lives." b. "It's true that your life may be different. What concerns you the most?" c. "It is really too early to know how much your life will be changed by the burn." d. "Why do you feel that way? You will be able to adapt as your recovery progresses."

ANS: B This response acknowledges the patient's feelings and asks for more assessment data that will help in developing an appropriate plan of care to assist the patient with the emotional response to the burn injury. The other statements are accurate, but do not acknowledge the anxiety and depression that the patient is expressing.

Esomeprazole (Nexium) is prescribed for a patient who incurred extensive burn injuries 5 days ago. Which nursing assessment would best evaluate the effectiveness of the medication? a. Bowel sounds b. Stool frequency c. Abdominal distention d. Stools for occult blood

ANS: D H2 blockers and proton pump inhibitors are given to prevent Curling's ulcer in the patient who has suffered burn injuries. Proton pump inhibitors usually do not affect bowel sounds, stool frequency, or appetite

A patient has experienced an electrical burn and has developed thick eschar over the burn site. Which of the following topical antibacterial agents will the nurse expect the physician to order for the wound? A) Silver sulfadiazine 1% (Silvadene) water-soluble cream B) Mafenide acetate 10% (Sulfamylon) hydrophilic-based cream C) Silver nitrate 0.5% aqueous solution D) Acticoat

Ans: B Feedback: Mafenide acetate 10% hydrophilic-based cream is the agent of choice when there is a need to penetrate thick eschar. Silver products do not penetrate eschar; Acticoat is a type of silver dressing.

A nurse is teaching a patient with a partial-thickness wound how to wear his elastic pressure garment. How would the nurse instruct the patient to wear this garment? A) 4 to 6 hours a day for 6 months B) During waking hours for 2 to 3 months after the injury C) Continuously D) At night while sleeping for a year after the injury

Ans: C Feedback: Elastic pressure garments are worn continuously (i.e., 23 hours a day).

While performing a patient's ordered wound care for the treatment of a burn, the patient has made a series of sarcastic remarks to the nurse and criticized her technique. How should the nurse best interpret this patient's behavior? A) The patient may be experiencing an adverse drug reaction that is affecting his cognition and behavior. B) The patient may be experiencing neurologic or psychiatric complications of his injuries. C) The patient may be experiencing inconsistencies in the care that he is being provided. D) The patient may be experiencing anger about his circumstances that he is deflecting toward the nurse.

Ans: D Feedback: The patient may experience feelings of anger. The anger may be directed outward toward those who escaped unharmed or toward those who are now providing care. While drug reactions, complications, and frustrating inconsistencies in care cannot be automatically ruled out, it is not uncommon for anger to be directed at caregivers.

A patient arrives in the emergency department after being burned in a house fire. The patient's burns cover the face and the left forearm. What extent of burns does the patient most likely have? A) 13% B) 25% C) 9% D) 18%

Ans: D Feedback: When estimating the percentage of body area or burn surface area that has been burned, the Rule of Nines is used: the face is 9%, and the forearm is 9% for a total of 18% in this patient.

Rule of nines for an adult?

Entire head and neck=10 Entire arm=9 Entire leg=18 anterior trunk=18 posterior trunk=18 perineum=1 Neck=1

Acute Phase Nursing and Collaborative Management (Cont'd)

Excision and grafting Eschar is removed down to the subcutaneous tissue or fascia Graft is placed on clean, viable tissue Wound is covered with autograft

Acute Phase Nursing and Collaborative Management

Excision and grafting Cultured epithelial autografts Grown from biopsies obtained from the patient's own skin Used in patients with a large body surface burn area or those with limited skin for harvesting

Priority ICU nurse care plan for a patient with severe full thickness and deep partial thickness burns over half of the body?

High risk for infection

Why do you never apply an ice pack to a burn?

Hinders blood flow- further damaging cells

Burn depth

Temperature and length of exposure

Family and patient support groups Special Needs of the Nursing Staff

The nurse cares for patients who, at times, may be unpleasant, hostile, apprehensive, and frustrated Nurses new to burn nursing often find it difficult to cope

Pastoral care Rehabilitation Phase

The rehabilitation phase is defined as beginning when The patient's burn wounds are covered with skin or healed The patient is able to resume a level of self-care activity

Interventions to help regulate body temperature when patient loses skin?

Turn up the temperature and keep the patient warm

Emergent Phase Complications (Cont'd)

Urinary system ↓ Blood flow to kidneys causes renal ischemia Acute tubular necrosis (ATN)

What do you use/not use to titrate urine output?

Use catheters. Dont use diuretics, steroids, or pressors.

What to use/not use to stop burning process?

Use cool water sparingly and only if necessary; dont use any topical meds

Complications of an acute burn event during treatment?

Watch for signs of stress ulcers

Third degree burn; Full thickness burn

White and leathery; can actually have many colors/appearances, no cap refill, hair follicles not intact, no pain sensation in areas of burn. Example: caused by flame, electric current, or chemical burns. Effects epidermis, entire dermis, and sometimes subcutaneous tissue, may involve caonnective tissue, muscle, and bone

A patient is admitted to the emergency department with first- and second-degree burns after being involved in a house fire. Which of the following assessment findings would alert you to the presence of an inhalation injury (select all that apply)? A.Singed nasal hair B.Generalized pallor C.Painful swallowing D.Burns on the upper extremities E.History of being involved in a large fire

a, c. Reliable clues to the occurrence of inhalation injury is the presence of facial burns, singed nasal hair, hoarseness, painful swallowing, darkened oral and nasal membranes, carbonaceous sputum, history of being burned in an enclosed space, and cherry red skin color

When assessing a pt with a partial thickness burn, the nurse would expect to find (select all that apply) a. blisters b. exposed fascia c. exposed muscles d. intact nerve endings e. red, shiny, wet appearance

a, d, e

The injury that is least likely to result in a full thickness burn is: a. sunburn b. scald injury c. chemical burn d. electrical injury

a.

The nurse is preparing to care for a burn client scheduled for an escharotomy procedure being performed for a third degree circumferential arm burn. The nurse understands that the anticipated therapeutic outcome of the escharotomy is: a. return of distal pulses b. brisk bleeding from the site c. decreasing edema formation d. formation of granulation tissue

a. Escharotomies arepreformed to relieve the compartment syndrome that can occur when edema forms under nondistensible eschar in a circumferential third degree burn.

A client's burn is infected and mafenide (Sulfamylon) is prescribed. The nurse's knowledge about this medication would indicate that which organism is involved? a. pseudomonas aeruginosa b. tubercle bacillus c. Methicillin resistant staphylococcus aureus (MRSA) d. Candida albicans

a. Mafenide is useful in treatment of partial and full thickness burns to prevent septicemia caused by organisms suche as pseudomonas aeruginosa.

A male burn pt who was struck by lightning arrives at the emergency department with full thickness burns to the arms and chest and with a cervical collar in place. Which assessment finding is the nurse's priority? a. serum K+ of 5.6 mEq/L b. Arterial blood ph of 7.35 c. Cervical spine fracture d. hemoglobin 18g/dL

a. The pt's potassium level puts them at risk for life threatening cardiac dysrhythmias.

A client is admitted to a burn intensive care unit with extensive full thickness burns. What should be the nurse's initial concern? a. fluid status b. risk for infection c. body image d. level of pain

a. in early burn care, the client's greatest need has to do with fluid resuscitation because of large volume fluid loss through the damaged skin.

A pt in the emergent phase of burn care for thermal burns on 20% of the total body surface area is unconscious. Which assessment data is the most important for the nurse's evaluation of the pt's injuries? a. condition of the oropharynx b. percentage of TBSA affected c. location of the pt in the fire d. comorbidities of the pt

a. the pt is likely to have suffered a smoke inhalation injury because thermal burns are caused by flames that emit smoke and because the pt is unconscious.

goal of the rehabilitation phase

assist patient in resuming functional roles

fluid therapy requires what?

at least 2 large bore IVs

When caring for a client with extensive burns, the nurse anticipates that pain medication will be administered via which route? a. oral b. IV c. IM d. Subq

b.

The nurse is caring for a patient with superficial partial-thickness burns of the face sustained within the last 12 hours. Upon assessment, the nurse would expect to find which of the following symptoms? A. Blisters B. Reddening of the skin C. Destruction of all skin layers D. Damage to sebaceous glands

b. The clinical appearance of superficial partial-thickness burns includes erythema, blanching with pressure, and pain and minimal swelling with no vesicles or blistering during the first 24 hours.

A pt who has an inhalation injury is receiving albuterol (Ventolin) for bronchospasm. What is the most important adverse effect of this medication for the nurse to manage? a. gi distress b. tachycardia c. restlessness d. hypokalemia

b. albuterol stimulates beta adrenergic receptors in the lungs to cause bronchodilation and is nonselective and also causes receptors in the heart to increase heart rate.

The nurse should expect to apply which type of ordered antiseptic to a client with a burn wound, once the area has been cleansed with sterile saline? a. copper containing b. silver containing c. biguanide d. acetic acid

b. silver sulfadiazine is a metallic type of antiseptic that is widely used on burns. The silver in the solution is toxic to bacteria, and prevents them from reproducing.

The nurse is administering fluids intravenously as prescribed to a client who sustained superficial partial thickness burn injuries of the back and legs. In evaluating the adequacy of fluid resuscitation, the nurse understands that which of the following would provide the most reliable indicator for determining the adequacy? a. vital signs b. urine output c. mental status d. peripheral pulses

b. successful or adequate fluid resuscitation in the client is signaled by stable vital signs, adequate urine output, palpable peripheral pulses and clear sensorium.

Knowing the most common causes of household fires, which of the following prevention strategies would the nurse focus on when teaching about fire safety? a. set how water temp at 140 F b. use only hardwired smoke detectors c. encourage regular home fire exit drills d. never permit older adults to cook unattended.

c.

When caring for a patient with an electrical burn injury, the nurse should question a health care provider's order for A. Mannitol 75 gm IV. B. Urine for myoglobulin. C. Lactated Ringer's at 25 ml/hr. D. Sodium bicarbonate 24 mEq every 4 hours.

c. Electrical injury puts the patient at risk for myoglobinuria, which can lead to acute renal tubular necrosis (ATN). Treatment consists of infusing lactated Ringer's at a rate sufficient to maintain urinary output at 75 to 100 ml/hr. Mannitol can also be used to maintain urine output. Sodium bicarbonate may be given to alkalinize the urine. The urine would also be monitored for the presence of myoglobin. An infusion rate of 25 ml/hr is not sufficient to maintain adequate urine output in prevention and treatment of ATN.

A child has just been admitted to the pediatric burn unit. Currently, the child is being evaluated for burns to his chest and upper legs. He complains of thirst and asks for a drink. What is the most appropriate nursing action? a. give a small glass of clear liquid b. give a small glass of a full liquid c. keep the child NPO d. order a pediatric meal tray with extra liquids

c. Until a complete assessment and treatment plan are initiated, the child should be kept NPO. A complication of major burns is paralytic ileus, so until that has been ruled out, oral fluids should not be provided.

A patient is admitted with first- and second-degree burns covering the face, neck, entire right upper extremity, and the right anterior trunk area. Using the rule of nines, the nurse would calculate the extent of these burns as being A. 9%. B. 18%. C. 22.5%. D. 36%.

c. Using the rule of nines, the face and neck together encompass 4.5% of the body area; the right upper arm encompasses 9% of the body area; and the entire anterior trunk encompasses 18% of the body area. Since the patient has burns on only the right side of the anterior trunk, the nurse would assess that burn as encompassing half of the 18%, or 9%. Therefore adding the three areas together, the nurse would correctly calculate the extent of this patient's burns to cover approximately 22.5% of the total body surface area.

A nurse is caring for a client with a new donor site that was harvested to treat a burn. The nurse should position the client to: a. allow ventilation of the site b. make the site dependent c. avoid pressure on the site d. keep the site fully covered

c. a universal concern in the care of donor sites for burn care is to keep the site away from sources of pressure.

The adult client was burned as a result of an explosion. The burn initially affected the client's entire face (anterior half of the head) and the upper half of the anterior torso, and there were circumferential burns to the lower half of both arms. The client's clothes caught on fire, and the client ran, causing subsequent burn injuries to the posterior surface of the head and the upper half of the posterior torso. Using the rule of nines, what would be the extent of the burn injury? a. 18% b. 24% c. 36% d. 48%

c. anterior head = 4.5%, upper half of anterior torso = 9%, lower half of both arms is 9%, posterior head 4.5%, upper half of posterior torso 9%, total 36%

The condition of a client with extensive third degree burns begins to deteriorate. The nurse is aware that which type of shock may occur as a result of inadequate circulating blood volume that occurs with a burn injury? a. cardiogenic b. distributive c. hypovolemic d. septic

c. burns and the resulting low circulating fluid volume can cause hypovolemic shock.

A client is undergoing fluid replacement after being burned on 20% of her body 12 hours ago. The nursing assessment reveals a blood pressure of 90/50, a pulse of 110, and urine output of 20 mL over the past hour. The nurse reports the findings to the physician and anticipates which of the following prescriptions? a. transfusing 1 unit of packed red blood cells b. administering a diuretic to increase urine output c. increasing the amount of IV lactated Ringers solution administered per hour d. changing the IV lactated Ringer's solution to one that contains dextrose in water.

c. fluid management during the first 24 hours following a burn injury generally includes the infusion of LR solution. Fluid resuscitation is determined by urine output and hourly urine output should be at least 30mL/hr. The client's urine output is indicative of insufficient fluid resuscitation, which places the client at risk for inadequate perfusion of the brain, heart, kidneys, and other body organs. Therefore, should expect ↑ of LR's.

The nurse manager is observing a new nursing graduate caring for a burn client in protective isolation. The nurse manager intervenes if the new nursing graduate planned to implement which incorrect component of protective isolation technique? a. using sterile sheets and linens b. performing strict hand washing technique c. wearing gloves and gown only when giving direct care to the client d. wearing protective garb, including a mask, gloves, cap, shoe covers, gowns, and plastic apron

c. ppe should be worn whenever entering the client's room

what are the most common injuries for a person struck by lightening?

cardiac and neurological

burns can cause complications with what body systems? (3)

cardiovascular, respiratory and urinary

A patient who is admitted to a burn unit is hypovolemic. A new nurse asks an experienced nurse about the patient's condition. Which response if made by the experienced nurse is most appropriate? a) "Blood loss from burned tissue is the most likely cause of hypovolemia." b) "Third spacing of fluid into fluid-filled vesicles is usually the cause of hypovolemia." c) "The usual cause of hypovolemia is vaporation of fluid from denuded body surfaces." d) "Increased capillary permeability causes fluid shifts out of blood vessels and results in hypovolemia."

d Rationale: Hypovolemic shock is caused by a massive shift of fluids out of the blood vessels as a result of increased capillary permeability. Water, sodium, and plasma proteins move into interstitial spaces and other surrounding tissue.

During the emergent phase of burn injury, the nurse assesses for the presence of hypovolemia. In burn patients, hypovolemia occurs primarily as a result of a. Blood loss from injured tissue. b. Third spacing of fluid into fluid-filled vesicles. c. Evaporation of fluid from denuded body surfaces. d. Capillary permeability with fluid shift to the interstitium.

d Rationale: Hypovolemic shock is caused by a massive shift of fluids out of the blood vessels as a result of increased capillary permeability. Water, sodium, and plasma proteins move into interstitial spaces and other surrounding tissue.

A victim of an industrial accident has chemical spilled on his face and body. The chemical, which has a pH of 7.51, is flushed with water by paramedics. What is the most important information for the receiving nurse to obtain about the pt from the paramedics? a. containment of chemical b. duration of water flushing c. other injuries of the victim d. specific location of accident

d. The nurse must know where the accident occurred to determine if the pt was rescued from an enclosed space. If so, the pt is at high risk for an inhalation injury because the enclosure concentrates the noxious fumes making an inhalation injury more likely.

A male pt suffered full thickness burns to the chest and back and the nurse notes the pressure alarm on his mechanical ventilator is sounding every 5 minutes. What is the most relevant assessment to prevent respiratory complications in this pt? a. pH b. PaCO2 c. Breath sounds d. chest expansion

d. assessing the pt's chest expansion is initially the most important because a sever burn that includes the anterior and posterior thorax can restrict chest expansion from eschar or scar tissue.

what is important to remember when wrapping up a burn

do not wrap when in a flexed position- can cause a contracture

if using a cold pack for a thermal burn what is important to remember?

to put a barrier between the skin and the cold pack

Complication within the neurological system during the acute phase

patient is usually ok- unless has respiratory distress and becomes hypoxic -this causes the patient to have a decrease in LOC

Complications within the endocrine system during the acute phase

patient may require insulin (the body is using up a lot of energy)- by keeping the glucose in control it may help to prevent infection

nursing management for psychosoical

social work consult, pastoral care consult, family support

chemical burns can cause?

systemic toxicity (can affect the entire body)

fourth degree burns

through muscles tendons and possibly to bone

TBSA for legs

18% (9% for front & 9% for back)

complications of burns- urinary (1)

-renal failure due to acute tubular necrosis

Fluid starting points for ages 14 and older?

500ml LR per hour

when do you want to administer the tetanus vaccination

BEFORE THE FLUID SHIFT OCCURS

How do you assess for possible inhalation injury?

Big risk for impaired gas exchange (look for soot around nasal passages) If present, client may be intubated before airway is compromised. -Hypovolemic shock -Infection

Acute Phase Complications (Cont'd)

Cardiovascular and respiratory systems Same complications can be present in the emergent phase and may continue into the acute phase

Emergent Phase Complications

Cardiovascular system Dysrhythmias and hypovolemic shock Edema Ischemia Necrosis Paresthesias Gangrene Impaired microcirculation and ↑ viscosity → sludging

Which type of burns need grafts?

Deep partial thickness and full thickness burns

Which route do you give meds?

IV

if the escharotomy does not work what do you do?

a fashotomy (fasciotomy)

TBSA for head and arms

9% (4.5% for front & 4.5% for back)

Full thickness to 40% of the body including both legs. Which measure should be instituted before transfer?

Adequate peripheral circulation to both feet

A patient comes into the ED fro extensive burns from a house fire, what is the priority assesment?

Airway then breathing

A patient is brought to the emergency department with a burn injury. The nurse knows that the first systemic event after a major burn injury is what? A) Hemodynamic instability B) Gastrointestinal hypermotility C) Respiratory arrest D) Hypokalemia

Ans: A Feedback: The initial systemic event after a major burn injury is hemodynamic instability, which results from loss of capillary integrity and a subsequent shift of fluid, sodium, and protein from the intravascular space into the interstitial spaces. This precedes GI changes. Respiratory arrest may or may not occur, largely depending on the presence or absence of smoke inhalation. Hypokalemia does not take place in the initial phase of recovery.

Complication of surgical debridement?

Bleeding

Which GI assessment occurs in half of the burn victims?

Curling ucler

Rehabilitation Phase Pathophysiologic Changes (Cont'd)

Discoloration of scar fades with time Pressure can help keep scar flat Newly healed areas can be hypersensitive or hyposensitive to cold, heat, and touch Healed areas must be protected from direct sunlight for 1 year

what would you provide to a patient if you suspect them of having an inhalation burn? (or carbon monoxide poising >10%?)

*a non-rebreather mask* (provides 100% O2 to the patient)

nursing management during acute phase (6)

*before and after aquacel ag burn dressing* -wound care -excision and grafting -pain management -PT and OT -nutrition -psychosocial

what should you be most concerned about what an electrical burn?

*heart damage* (always assume that there is something more wrong than just trauma)

nose and ears are at a risk for?

*infection* because the cartilage takes longer to heal -not great blood flow

what is the priority of an inhalation burn?

*priority- rapid respiratory assessment* (want to ensure that there is no airway issues)

nursing management during rehabilitation phase

*psychosocial needs huge!!!* -patient may have bad disabilities and can no longer do what they use to be able to -body image issues- can be disfiguring -family support -self-esteem issues -PTSD

if there is a burn on the chest or face what is the person at risk for? and why??

*respiratory distress* -the fluid shift causes a lot of edema and swelling which can constrict the airway

what should a burn patient avoid during the healing process

*sun exposure in affected area* -skin is very sensitive and can easily become burned -want to put on sunscreen everyday- even under clothes

*the main goal during care for burns is*

*to prevent hypovolemic shock and edema formation*

when does the 24 hours begin?

- 24 hr starts at TIME OF BURN -if they are an hour after burn, then you have 7 hr to get in the 1/2

#1 complication faced during the acute phase

-*INFECTION* -number one cause of death - if need to be on an antibiotic- put on a topical antibiotic -not IV unless septic

acute phase is focused on

-*focus is on infection control, wound care* -fluid and electrolyte balance (sodium and potassium)

TBSA

-*rule of 9s used for quick assessment* -*use palm for 1% estimate of splotchy spots*

ideal urinary output for burn patients -how much urine per mL/kg/hr -adults (mL/hr) -children (mL/kg/hr) -electrical burn patients for adults and children

-0.5-1 mL of urine/kg/hr -adults: 30-50 mL/ hr -children: 1 mL/kg/hr -electrical injury: adults 100 mL/ hr -electrical injury: children 2 mL/kg/hr

nursing assessment of burns (7)

-ABCs -VS -EKG rhythm (tall T waves) -LOC -decreased BS (illeus possible) -shivering (heat and volume loss, anxiety, pain) -shock

burns going through the healing process become very itchy- what would you tell the patient?

-DO NOT SCRATCH -provide moisturizer to help prevent itching

additional complications for burns (2)

-DVT -pneumonia from inhalation injuries

where should the IV go in a burn victim

-IV in non-burned arm or assist with central line placement

when does the acute phase begin and end

-begins: when the patient is hemodynamically stable & capillary permiability is restored -usually about 48-72 hours after burn -ends: when the burn is completely covered with a skin graft or the wound is completely healed

nursing management for excision and grafting

-best to use own skin- prevents rejection and infection -donor site will heal within 10 days -skin usually taken from butt or top of thigh -can use a cadaver graft or artificial skin graft -will be covered with tegaderm- *normal to see serosanganus pooling in the tegaderm*

electrical burns

-burns are caused by heat generated by electrical energy as it passes through the body -*results in internal tissue damage* -the voltage, type of current, length of contact, contact site, and duration of contact are important to identify -lightening: acts as massive defibrillation depolarizing myocardium and producing period of asystole, cardiac and neurological injuries are most frequent

chemical burns

-burns are caused by tissue contact with strong acids, alkalis, or organic compounds -cutaneous absorption can occur -can be from splash like chemical eye splash (need eye rinse stations)

when is pressure garment therapy used?

-can be worn after the burns are healed -worn 23/24 hours of the day -helps to compress scars and flatten them out and prevent contractures

third degree burns

-charred and whitish area -leathery appearance -skin becomes dry and hard -all skin elements destroyed -not as painful as second degree- because the nerve is destroyed

the most common type of fluid that is used for replacement (2)

-colloids (albumin) -LR (crystalloid)

severity of burns is determined by (4)

-depth -extent of total body surface area (TBSA) -location (chest, face, neck bad) -patient risk factors

emergent (resusitative) phase

-direct burn injury to vessels increases capillary permeability -onset to time of injury till 48- 72 hr; ends when fluid mobilization and diuresis starts -includes prehospital care and ER care -prevent hypovolemic shock and edema formation

complications that can occur from electrical burns (4)

-dysrhythmias -cardiac arrest -trauma (broken bones, head injury, spinal injury) -actue renal failure

complications of burns- cardiovascular (3)

-dysrhythmias -hypovolemic shock -compartment syndrome

burns of face or head

-elevate HOB 30 degrees or more -watch for respiratory -go to burn center -intubated until fluid shift resolves -anything more than 10% automatic referral to burn center -stage it after debriding

signs of an inhalation burn (8)

-facial injury or burn -singed nasal hairs -black sputum (carbonaceous) -wheezing -pharyngeal edema -stridor breath sounds (high pitch) -hoarseness to voice -painful swallowing

thermal burns

-flame, flash, scald, contact with hot object -most common type -touched something hot, hot water -examples: curling iron burn, sun burn

interventions to promote healing on the face and arms?

-for the face- limit pressure -elevate the arms

nursing management for respiratory and airway

-give O2- 100% *nonrebreather* (any inhalation injury or carbon monoxide) -intubate quickly before swelling and requires a trach -HOB elevated to 30 degrees

location- circumferential burns

-go all the way around, fluid shift and swelling happens -will close off blood vessels and cut off circulation -assess pulses and cap refull -can happen on abdomen as well -compartment syndrome

caring for the nurse (4)

-highly emotionally demanding -tremendous care burden -burn out common -nurturing new staff and old staff is necessary

inhalation burns

-inhalation of hot air or noxious chemicals -above the glottis- upper airway-hot air, smoke, steam -below the glottis- lower airway- usually chemical -closed space injury

nursing actions for electrical burns

-look for entry and exit wounds (tells you the pathway) -*stabilize the cervical spine* (especially if head/neck trauma) -provide O2 -*EKG monitoring*

nursing management for fluid therapy

-losing a lot of fluid and want to give back the amount of fluid they are losing -parkland formula determines how much fluid the patient is going to need -see how well they are responding to fluid by checking *urinary output* -fluid replacement therapy should happen ASAP (ideally before arriving to ER, but if not then immediately need IV) -if burned so bad can't get IV- need central line stitched in *NEED TO HAVE TO HAVE ACCESS*

carbon monoxide poisoning

-majority of deaths a fire scene -displaces O2 on hemoglobin-hypoxemia- death if levels > 20% -over 10% need nonrebreather mask -over 20% will not survive

second degree burns

-more deep partial -goes into subQ tissue -*BLISTERING* -swelling-mild to moderate -more of the time heal without skin graft -if healing is prolonged- skin graft -heals 10 to 21 days

wound debridement

-once patient is stable -necrotic skin removed -painful (needs to be sedated or under anesthesia) -may have escharotomies and fasciotomies -dressings: aquacel Ag (silver- antimicrobial), Mepliex Ag (padding with silver) -extremity needs to be elevated -ROM can help but not to soon- can prevent contractures -DO NOT put burn area in flexed position

nutritional needs

-promote wound healing, decrease mortality -basal metabolic rate is high and caloric needs are greater -NPO status until bowel sounds are heard -enteral feedings or TPN (if no bowel sounds) -high protein, CHO, fats, vitamins

first degree burns

-red skin but no scaring -painful -do not count it when determining body surface area of burns -heals fast

prehospital phase (12) (starts at scene of action)

-remove source of burn -ABCs -O2 -remove burned clothing if possible (esp chemical) -cover with clean, dry sheet -inhalation- watch respiratory -IV accesss -begin fluid replacement -possible foley -elevate burned limbs -pain management -check for any other injuries

complications of burns- respiratory (2)

-respiratory distress -pulmonary edema (can be caused by giving fluids too fast)(need to listen to lung and heart sounds)

escharotomy

-similar to fasciotomy but *more superficial* -done to improve circulation -by scalpel or cautery -wounds they make are packed with fine mesh gauze -lengthwise incision is made through the burn eschar to relieve constriction and pressure and improve circulation -performed at bedside without anesthesia, or OR -nerve endings destroyed by burns so does not hurt -gives swelling room

burn prevention (5)

-smoke detectors -carbon monoxide detectors -fire extinguishers -fire drills -safety with combustibles, candles, water, heat, child proofing

fluid resuscitation is evaluated by? (4)

-stable vital signs -adequate urinary output -clear sensorium -palpable pulses

types of burns (5)

-thermal (most common) -chemical -inhalation -electrical -cold thermal

nursing management for pain management

-very important -want to treat before do wound care because that it very painful -IV meds are ideal -if no IV meds- PO are ok -always want to try and treat with non-pharmacological pain treatments first

fluid and electrolyte shifts

-watch for hypovolemic shock -2nd spacing or 3rd spacing -hyponatremia and hyperkalemia occur initially -fluid escpaes vessels and into tissues -second degree spacing or third spacing- where fluid shifts to within tissue itself

rehabilitation phase

-wounds are healed or covered with grafts -want patients to have self-care -goal can be reached as early as 2 weeks or a long as 2-3 and up to 7 months -ends with independent patient who has had cosmetic reconstructions complete (no further medical interventions planned)

TBSA for perineum

1%

O2 for inhalation injury?

100% via nonrebreather

TBSA for abdomen

36% (18% for front & 18% for back)

parkland formula

4 mL of LR x % TBSA x kg of weight -1/2 in first 8 hr -1/4 in second 8 -1/4 in third 8

An 80-kg patient with burns over 30% of total body surface area (TBSA) is admitted to the burn unit. Using the Parkland formula of 4 mL/kg/%TBSA, what is the IV infusion rate (mL/hour) for lactated Ringer's solution that the nurse will administer during the first 8 hours?

600 mL The Parkland formula states that patients should receive 4 mL/kg/%TBSA burned during the first 24 hours. Half of the total volume is given in the first 8 hours and then the last half is given over 16 hours: 4 80 30 = 9600 mL total volume; 9600/2 = 4800 mL in the first 8 hours; 4800 mL/8 hr = 600 mL/hr.

SHORT ANSWER The nurse estimates the extent of a burn using the rule of nines for a patient who has been admitted with deep partial-thickness burns of the anterior trunk and the entire left arm. What percentage of the patient's total body surface area (TBSA) has been injured?

ANS: 27% When using the rule of nines, the anterior trunk is considered to cover 18% of the patient's body and each arm is 9%.

SHORT ANSWER An 80-kg patient with burns over 30% of total body surface area (TBSA) is admitted to the burn unit. Using the Parkland formula of 4 mL/kg/%TBSA, what is the IV infusion rate (mL/hour) for lactated Ringer's solution that the nurse will administer during the first 8 hours?

ANS: 600 mL The Parkland formula states that patients should receive 4 mL/kg/%TBSA burned during the first 24 hours. Half of the total volume is given in the first 8 hours and then the last half is given over 16 hours: 4 × 80 × 30 = 9600 mL total volume; 9600/2 = 4800 mL in the first 8 hours; 4800 mL/8 hr = 600 mL/hr.

In which order will the nurse take these actions when doing a dressing change for a partial-thickness burn wound on a patient's chest? (Put a comma and a space between each answer choice [A, B, C, D, E].) a. Apply sterile gauze dressing. b. Document wound appearance. c. Apply silver sulfadiazine cream. d. Administer IV fentanyl (Sublimaze). e. Clean wound with saline-soaked gauze

ANS: D, E, C, A, B Because partial-thickness burns are very painful, the nurse's first action should be to administer pain medications. The wound will then be cleaned, antibacterial cream applied, and covered with a new sterile dressing. The last action should be to document the appearance of the wound.

A nurse is caring for a patient in the emergent/resuscitative phase of burn injury. During this phase, the nurse should monitor for evidence of what alteration in laboratory values? A) Sodium deficit B) Decreased prothrombin time (PT) C) Potassium deficit D) Decreased hematocrit

ANS: A Feedback: Anticipated fluid and electrolyte changes that occur during the emergent/resuscitative phase of burn injury include sodium deficit, potassium excess, base-bicarbonate deficit, and elevated hematocrit. PT does not typically decrease.

A patient arrives in the emergency department with facial and chest burns caused by a house fire. Which action should the nurse take first? a. Auscultate the patient's lung sounds. b. Determine the extent and depth of the burns. c. Infuse the ordered lactated Ringer's solution. d. Administer the ordered hydromorphone (Dilaudid).

ANS: A A patient with facial and chest burns is at risk for inhalation injury, and assessment of airway and breathing is the priority. The other actions will be completed after airway management is assured.

Which patient is most appropriate for the burn unit charge nurse to assign to a registered nurse (RN) who has floated from the hospital medical unit? a. A 34-year-old patient who has a weight loss of 15% from admission and requires enteral feedings. b. A 67-year-old patient who has blebs under an autograft on the thigh and has an order for bleb aspiration c. A 46-year-old patient who has just come back to the unit after having a cultured epithelial autograft to the chest d. A 65-year-old patient who has twice-daily burn debridements and dressing changes to partial-thickness facial burns

ANS: A An RN from a medical unit would be familiar with malnutrition and with administration and evaluation of response to enteral feedings. The other patients require burn assessment and care that is more appropriate for staff who regularly care for burned patients.

A patient has just been admitted with a 40% total body surface area (TBSA) burn injury. To maintain adequate nutrition, the nurse should plan to take which action? a. Insert a feeding tube and initiate enteral feedings. b. Infuse total parenteral nutrition via a central catheter. c. Encourage an oral intake of at least 5000 kcal per day. d. Administer multiple vitamins and minerals in the IV solution.

ANS: A Enteral feedings can usually be initiated during the emergent phase at low rates and increased over 24 to 48 hours to the goal rate. During the emergent phase, the patient will be unable to eat enough calories to meet nutritional needs and may have a paralytic ileus that prevents adequate nutrient absorption. Vitamins and minerals may be administered during the emergent phase, but these will not assist in meeting the patient's caloric needs. Parenteral nutrition increases the infection risk, does not help preserve gastrointestinal function, and is not routinely used in burn patients.

The charge nurse observes the following actions being taken by a new nurse on the burn unit. Which action by the new nurse would require an intervention by the charge nurse? a. The new nurse uses clean latex gloves when applying antibacterial cream to a burn wound. b. The new nurse obtains burn cultures when the patient has a temperature of 95.2° F (35.1° C). c. The new nurse administers PRN fentanyl (Sublimaze) IV to a patient 5 minutes before a dressing change. d. The new nurse calls the health care provider for a possible insulin order when a nondiabetic patient's serum glucose is elevated.

ANS: A Sterile gloves should be worn when applying medications or dressings to a burn. Hypothermia is an indicator of possible sepsis, and cultures are appropriate. Nondiabetic patients may require insulin because stress and high calorie intake may lead to temporary hyperglycemia. Fentanyl peaks 5 minutes after IV administration, and should be used just before and during dressing changes for pain management

The charge nurse observes the following actions being taken by a new nurse on the burn unit. Which action by the new nurse would require an intervention by the charge nurse? a. The new nurse uses clean latex gloves when applying antibacterial cream to a burn wound. b. The new nurse obtains burn cultures when the patient has a temperature of 95.2° F (35.1° C). c. The new nurse administers PRN fentanyl (Sublimaze) IV to a patient 5 minutes before a dressing change. d. The new nurse calls the health care provider for a possible insulin order when a nondiabetic patient's serum glucose is elevated.

ANS: A Sterile gloves should be worn when applying medications or dressings to a burn. Hypothermia is an indicator of possible sepsis, and cultures are appropriate. Nondiabetic patients may require insulin because stress and high calorie intake may lead to temporary hyperglycemia. Fentanyl peaks 5 minutes after IV administration, and should be used just before and during dressing changes for pain management.

A patient with circumferential burns of both legs develops a decrease in dorsalis pedis pulse strength and numbness in the toes. Which action should the nurse take? a. Notify the health care provider. b. Monitor the pulses every 2 hours. c. Elevate both legs above heart level with pillows. d. Encourage the patient to flex and extend the toes on both feet.

ANS: A The decrease in pulse in a patient with circumferential burns indicates decreased circulation to the legs and the need for an escharotomy. Monitoring the pulses is not an adequate response to the decrease in circulation. Elevating the legs or increasing toe movement will not improve the patient's circulation

A patient with circumferential burns of both legs develops a decrease in dorsalis pedis pulse strength and numbness in the toes. Which action should the nurse take? a. Notify the health care provider. b. Monitor the pulses every 2 hours. c. Elevate both legs above heart level with pillows. d. Encourage the patient to flex and extend the toes on both feet.

ANS: A The decrease in pulse in a patient with circumferential burns indicates decreased circulation to the legs and the need for an escharotomy. Monitoring the pulses is not an adequate response to the decrease in circulation. Elevating the legs or increasing toe movement will not improve the patient's circulation.

The nurse caring for a patient admitted with burns over 30% of the body surface assesses that urine output has dramatically increased. Which action by the nurse would best ensure adequate kidney function? a. Continue to monitor the urine output. b. Monitor for increased white blood cells (WBCs). c. Assess that blisters and edema have subsided. d. Prepare the patient for discharge from the burn unit.

ANS: A The patient's urine output indicates that the patient is entering the acute phase of the burn injury and moving on from the emergent stage. At the end of the emergent phase, capillary permeability normalizes and the patient begins to diurese large amounts of urine with a low specific gravity. Although this may occur at about 48 hours, it may be longer in some patients. Blisters and edema begin to resolve, but this process requires more time. White blood cells may increase or decrease, based on the patient's immune status and any infectious processes. The WBC count does not indicate kidney function. The patient will likely remain in the burn unit during the acute stage of burn injury

The nurse caring for a patient admitted with burns over 30% of the body surface assesses that urine output has dramatically increased. Which action by the nurse would best ensure adequate kidney function? a. Continue to monitor the urine output. b. Monitor for increased white blood cells (WBCs). c. Assess that blisters and edema have subsided. d. Prepare the patient for discharge from the burn unit.

ANS: A The patient's urine output indicates that the patient is entering the acute phase of the burn injury and moving on from the emergent stage. At the end of the emergent phase, capillary permeability normalizes and the patient begins to diurese large amounts of urine with a low specific gravity. Although this may occur at about 48 hours, it may be longer in some patients. Blisters and edema begin to resolve, but this process requires more time. White blood cells may increase or decrease, based on the patient's immune status and any infectious processes. The WBC count does not indicate kidney function. The patient will likely remain in the burn unit during the acute stage of burn injury.

An employee spills industrial acids on both arms and legs at work. What is the priority action that the occupational health nurse at the facility should take? a. Remove nonadherent clothing and watch. b. Apply an alkaline solution to the affected area. c. Place cool compresses on the area of exposure. d. Cover the affected area with dry, sterile dressings.

ANS: A With chemical burns, the initial action is to remove the chemical from contact with the skin as quickly as possible. Remove nonadherent clothing, shoes, watches, jewelry, glasses, or contact lenses (if face was exposed). Flush chemical from wound and surrounding area with copious amounts of saline solution or water. Covering the affected area or placing cool compresses on the area will leave the chemical in contact with the skin. Application of an alkaline solution is not recommended.

A patient is admitted to the burn unit with burns to the head, face, and hands. Initially, wheezes are heard, but an hour later, the lung sounds are decreased and no wheezes are audible. What is the best action for the nurse to take? a. Encourage the patient to cough and auscultate the lungs again. b. Notify the health care provider and prepare for endotracheal intubation. c. Document the results and continue to monitor the patient's respiratory rate. d. Reposition the patient in high-Fowler's position and reassess breath sounds.

ANS: B The patient's history and clinical manifestations suggest airway edema and the health care provider should be notified immediately, so that intubation can be done rapidly. Placing the patient in a more upright position or having the patient cough will not address the problem of airway edema. Continuing to monitor is inappropriate because immediate action should occur

A patient is admitted to the burn unit with burns to the head, face, and hands. Initially, wheezes are heard, but an hour later, the lung sounds are decreased and no wheezes are audible. What is the best action for the nurse to take? a. Encourage the patient to cough and auscultate the lungs again. b. Notify the health care provider and prepare for endotracheal intubation. c. Document the results and continue to monitor the patient's respiratory rate. d. Reposition the patient in high-Fowler's position and reassess breath sounds.

ANS: B The patient's history and clinical manifestations suggest airway edema and the health care provider should be notified immediately, so that intubation can be done rapidly. Placing the patient in a more upright position or having the patient cough will not address the problem of airway edema. Continuing to monitor is inappropriate because immediate action should occur.

A nurse is caring for a patient who has burns of the ears, head, neck, and right arm and hand. The nurse should place the patient in which position? a. Place the right arm and hand flexed in a position of comfort. b. Elevate the right arm and hand on pillows and extend the fingers. c. Assist the patient to a supine position with a small pillow under the head. d. Position the patient in a side-lying position with rolled towel under the neck.

ANS: B The right hand and arm should be elevated to reduce swelling and the fingers extended to avoid flexion contractures (even though this position may not be comfortable for the patient). The patient with burns of the ears should not use a pillow for the head because this will put pressure on the ears, and the pillow may stick to the ears. Patients with neck burns should not use a pillow because the head should be maintained in an extended position in order to avoid contractures.

While the patient's full-thickness burn wounds to the face are exposed, what is the best nursing action to prevent cross contamination? a. Use sterile gloves when removing old dressings. b. Wear gowns, caps, masks, and gloves during all care of the patient. c. Administer IV antibiotics to prevent bacterial colonization of wounds. d. Turn the room temperature up to at least 70° F (20° C) during dressing changes.

ANS: B Use of gowns, caps, masks, and gloves during all patient care will decrease the possibility of wound contamination for a patient whose burns are not covered. When removing contaminated dressings and washing the dirty wound, use nonsterile, disposable gloves. The room temperature should be kept at approximately 85° F for patients with open burn wounds to prevent shivering. Systemic antibiotics are not well absorbed into deep burns because of the lack of circulation.

When assessing a patient who spilled hot oil on the right leg and foot, the nurse notes that the skin is dry, pale, hard skin. The patient states that the burn is not painful. What term would the nurse use to document the burn depth? a. First-degree skin destruction b. Full-thickness skin destruction c. Deep partial-thickness skin destruction d. Superficial partial-thickness skin destruction

ANS: B With full-thickness skin destruction, the appearance is pale and dry or leathery and the area is painless because of the associated nerve destruction. Erythema, swelling, and blisters point to a deep partial-thickness burn. With superficial partial-thickness burns, the area is red, but no blisters are present. First-degree burns exhibit erythema, blanching, and pain

When assessing a patient who spilled hot oil on the right leg and foot, the nurse notes that the skin is dry, pale, hard skin. The patient states that the burn is not painful. What term would the nurse use to document the burn depth? a. First-degree skin destruction b. Full-thickness skin destruction c. Deep partial-thickness skin destruction d. Superficial partial-thickness skin destruction

ANS: B With full-thickness skin destruction, the appearance is pale and dry or leathery and the area is painless because of the associated nerve destruction. Erythema, swelling, and blisters point to a deep partial-thickness burn. With superficial partial-thickness burns, the area is red, but no blisters are present. First-degree burns exhibit erythema, blanching, and pain.

A patient with burns covering 40% total body surface area (TBSA) is in the acute phase of burn treatment. Which snack would be best for the nurse to offer to this patient? a. Bananas b. Orange gelatin c. Vanilla milkshake d. Whole grain bagel

ANS: C A patient with a burn injury needs high protein and calorie food intake, and the milkshake is the highest in these nutrients. The other choices are not as nutrient-dense as the milkshake. Gelatin is likely high in sugar. The bagel is a good carbohydrate choice, but low in protein. Bananas are a good source of potassium, but are not high in protein and calories.

A patient with extensive electrical burn injuries is admitted to the emergency department. Which prescribed intervention should the nurse implement first? a. Assess oral temperature. b. Check a potassium level. c. Place on cardiac monitor. d. Assess for pain at contact points.

ANS: C After an electrical burn, the patient is at risk for fatal dysrhythmias and should be placed on a cardiac monitor. Assessing the oral temperature is not as important as assessing for cardiac dysrhythmias. Checking the potassium level is important. However, it will take time before the laboratory results are back. The first intervention is to place the patient on a cardiac monitor and assess for dysrhythmias, so that they can be treated if occurring. A decreased or increased potassium level will alert the nurse to the possibility of dysrhythmias. The cardiac monitor will alert the nurse immediately of any dysrhythmias. Assessing for pain is important, but the patient can endure pain until the cardiac monitor is attached. Cardiac dysrhythmias can be lethal.

A patient has just arrived in the emergency department after an electrical burn from exposure to a high-voltage current. What is the priority nursing assessment? a. Oral temperature b. Peripheral pulses c. Extremity movement d. Pupil reaction to light

ANS: C All patients with electrical burns should be considered at risk for cervical spine injury, and assessments of extremity movement will provide baseline data. The other assessment data are also necessary but not as essential as determining the cervical spine status

A patient has just arrived in the emergency department after an electrical burn from exposure to a high-voltage current. What is the priority nursing assessment? a. Oral temperature b. Peripheral pulses c. Extremity movement d. Pupil reaction to light

ANS: C All patients with electrical burns should be considered at risk for cervical spine injury, and assessments of extremity movement will provide baseline data. The other assessment data are also necessary but not as essential as determining the cervical spine status.

A patient with severe burns has crystalloid fluid replacement ordered using the Parkland formula. The initial volume of fluid to be administered in the first 24 hours is 30,000 mL. The initial rate of administration is 1875 mL/hr. After the first 8 hours, what rate should the nurse infuse the IV fluids? a. 350 mL/hour b. 523 mL/hour c. 938 mL/hour d. 1250 mL/hour

ANS: C Half of the fluid replacement using the Parkland formula is administered in the first 8 hours and the other half over the next 16 hours. In this case, the patient should receive half of the initial rate, or 938 mL/hr.

The nurse is reviewing laboratory results on a patient who had a large burn 48 hours ago. Which result requires priority action by the nurse? a. Hematocrit 53% b. Serum sodium 147 mEq/L c. Serum potassium 6.1 mEq/L d. Blood urea nitrogen 37 mg/dL

ANS: C Hyperkalemia can lead to fatal dysrhythmias and indicates that the patient requires cardiac monitoring and immediate treatment to lower the potassium level. The other laboratory values are also abnormal and require changes in treatment, but they are not as immediately life threatening as the elevated potassium level

The nurse is reviewing laboratory results on a patient who had a large burn 48 hours ago. Which result requires priority action by the nurse? a. Hematocrit 53% b. Serum sodium 147 mEq/L c. Serum potassium 6.1 mEq/L d. Blood urea nitrogen 37 mg/dL

ANS: C Hyperkalemia can lead to fatal dysrhythmias and indicates that the patient requires cardiac monitoring and immediate treatment to lower the potassium level. The other laboratory values are also abnormal and require changes in treatment, but they are not as immediately life threatening as the elevated potassium level.

On admission to the burn unit, a patient with an approximate 25% total body surface area (TBSA) burn has the following initial laboratory results: Hct 58%, Hgb 18.2 mg/dL (172 g/L), serum K+ 4.9 mEq/L (4.8 mmol/L), and serum Na+ 135 mEq/L (135 mmol/L). Which action will the nurse anticipate taking now? a. Monitor urine output every 4 hours. b. Continue to monitor the laboratory results. c. Increase the rate of the ordered IV solution. d. Type and crossmatch for a blood transfusion.

ANS: C The patient's laboratory data show hemoconcentration, which may lead to a decrease in blood flow to the microcirculation unless fluid intake is increased. Because the hematocrit and hemoglobin are elevated, a transfusion is inappropriate, although transfusions may be needed after the emergent phase once the patient's fluid balance has been restored. On admission to a burn unit, the urine output would be monitored more often than every 4 hours; likely every1 hour.

A patient who has burns on the arms, legs, and chest from a house fire has become agitated and restless 8 hours after being admitted to the hospital. Which action should the nurse take first? a. Stay at the bedside and reassure the patient. b. Administer the ordered morphine sulfate IV. c. Assess orientation and level of consciousness. d. Use pulse oximetry to check the oxygen saturation.

ANS: D Agitation in a patient who may have suffered inhalation injury might indicate hypoxia, and this should be assessed by the nurse first. Administration of morphine may be indicated if the nurse determines that the agitation is caused by pain. Assessing level of consciousness and orientation is also appropriate but not as essential as determining whether the patient is hypoxemic. Reassurance is not helpful to reduce agitation in a hypoxemic patient

A patient who has burns on the arms, legs, and chest from a house fire has become agitated and restless 8 hours after being admitted to the hospital. Which action should the nurse take first? a. Stay at the bedside and reassure the patient. b. Administer the ordered morphine sulfate IV. c. Assess orientation and level of consciousness. d. Use pulse oximetry to check the oxygen saturation.

ANS: D Agitation in a patient who may have suffered inhalation injury might indicate hypoxia, and this should be assessed by the nurse first. Administration of morphine may be indicated if the nurse determines that the agitation is caused by pain. Assessing level of consciousness and orientation is also appropriate but not as essential as determining whether the patient is hypoxemic. Reassurance is not helpful to reduce agitation in a hypoxemic patient.

Which action will the nurse include in the plan of care for a patient in the rehabilitation phase after a burn injury to the right arm and chest? a. Keep the right arm in a position of comfort. b. Avoid the use of sustained-release narcotics. c. Teach about the purpose of tetanus immunization. d. Apply water-based cream to burned areas frequently.

ANS: D Application of water-based emollients will moisturize new skin and decrease flakiness and itching. To avoid contractures, the joints of the right arm should be positioned in an extended position, which is not the position of comfort. Patients may need to continue the use of opioids during rehabilitation. Tetanus immunization would have been given during the emergent phase of the burn injury

Which action will the nurse include in the plan of care for a patient in the rehabilitation phase after a burn injury to the right arm and chest? a. Keep the right arm in a position of comfort. b. Avoid the use of sustained-release narcotics. c. Teach about the purpose of tetanus immunization. d. Apply water-based cream to burned areas frequently.

ANS: D Application of water-based emollients will moisturize new skin and decrease flakiness and itching. To avoid contractures, the joints of the right arm should be positioned in an extended position, which is not the position of comfort. Patients may need to continue the use of opioids during rehabilitation. Tetanus immunization would have been given during the emergent phase of the burn injury.

Esomeprazole (Nexium) is prescribed for a patient who incurred extensive burn injuries 5 days ago. Which nursing assessment would best evaluate the effectiveness of the medication? a. Bowel sounds b. Stool frequency c. Abdominal distention d. Stools for occult blood

ANS: D H2 blockers and proton pump inhibitors are given to prevent Curling's ulcer in the patient who has suffered burn injuries. Proton pump inhibitors usually do not affect bowel sounds, stool frequency, or appetite.

The nurse is reviewing the medication administration record (MAR) on a patient with partial-thickness burns. Which medication is best for the nurse to administer before scheduled wound debridement? a. Ketorolac (Toradol) b. Lorazepam (Ativan) c. Gabapentin (Neurontin) d. Hydromorphone (Dilaudid)

ANS: D Opioid pain medications are the best choice for pain control. The other medications are used as adjuvants to enhance the effects of opioids.

A patient who was found unconscious in a burning house is brought to the emergency department by ambulance. The nurse notes that the patient's skin color is bright red. Which action should the nurse take first? a. Insert two large-bore IV lines. b. Check the patient's orientation. c. Assess for singed nasal hair and dark oral mucous membranes. d. Place the patient on 100% oxygen using a non-rebreather mask.

ANS: D The patient's history and skin color suggest carbon monoxide poisoning, which should be treated by rapidly starting oxygen at 100%. The other actions can be taken after the action to correct gas exchange.

Eight hours after a thermal burn covering 50% of a patient's total body surface area (TBSA) the nurse assesses the patient. Which information would be a priority to communicate to the health care provider? a. Blood pressure is 95/48 per arterial line. b. Serous exudate is leaking from the burns. c. Cardiac monitor shows a pulse rate of 108. d. Urine output is 20 mL per hour for the past 2 hours.

ANS: D The urine output should be at least 0.5 to 1.0 mL/kg/hr during the emergent phase, when the patient is at great risk for hypovolemic shock. The nurse should notify the health care provider because a higher IV fluid rate is needed. BP during the emergent phase should be greater than 90 systolic, and the pulse rate should be less than 120. Serous exudate from the burns is expected during the emergent phase

Eight hours after a thermal burn covering 50% of a patient's total body surface area (TBSA) the nurse assesses the patient. Which information would be a priority to communicate to the health care provider? a. Blood pressure is 95/48 per arterial line. b. Serous exudate is leaking from the burns. c. Cardiac monitor shows a pulse rate of 108. d. Urine output is 20 mL per hour for the past 2 hours.

ANS: D The urine output should be at least 0.5 to 1.0 mL/kg/hr during the emergent phase, when the patient is at great risk for hypovolemic shock. The nurse should notify the health care provider because a higher IV fluid rate is needed. BP during the emergent phase should be greater than 90 systolic, and the pulse rate should be less than 120. Serous exudate from the burns is expected during the emergent phase.

A young adult patient who is in the rehabilitation phase after having deep partial-thickness face and neck burns has a nursing diagnosis of disturbed body image. Which statement by the patient indicates that the problem is resolving? a. "I'm glad the scars are only temporary." b. "I will avoid using a pillow, so my neck will be OK." c. "I bet my boyfriend won't even want to look at me anymore." d. "Do you think dark beige makeup foundation would cover this scar on my cheek?"

ANS: D The willingness to use strategies to enhance appearance is an indication that the disturbed body image is resolving. Expressing feelings about the scars indicates a willingness to discuss appearance, but not resolution of the problem. Because deep partial-thickness burns leave permanent scars, a statement that the scars are temporary indicates denial rather than resolution of the problem. Avoiding using a pillow will help prevent contractures, but it does not address the problem of disturbed body image

A young adult patient who is in the rehabilitation phase after having deep partial-thickness face and neck burns has a nursing diagnosis of disturbed body image. Which statement by the patient indicates that the problem is resolving? a. "I'm glad the scars are only temporary." b. "I will avoid using a pillow, so my neck will be OK." c. "I bet my boyfriend won't even want to look at me anymore." d. "Do you think dark beige makeup foundation would cover this scar on my cheek?"

ANS: D The willingness to use strategies to enhance appearance is an indication that the disturbed body image is resolving. Expressing feelings about the scars indicates a willingness to discuss appearance, but not resolution of the problem. Because deep partial-thickness burns leave permanent scars, a statement that the scars are temporary indicates denial rather than resolution of the problem. Avoiding using a pillow will help prevent contractures, but it does not address the problem of disturbed body image.

During the emergent phase of burn care, which assessment will be most useful in determining whether the patient is receiving adequate fluid infusion? a. Check skin turgor. b. Monitor daily weight. c. Assess mucous membranes. d. Measure hourly urine output.

ANS: D When fluid intake is adequate, the urine output will be at least 0.5 to 1 mL/kg/hour. The patient's weight is not useful in this situation because of the effects of third spacing and evaporative fluid loss. Mucous membrane assessment and skin turgor also may be used, but they are not as adequate in determining that fluid infusions are maintaining adequate perfusion.

A public health nurse has reviewed local data about the incidence and prevalence of burn injuries in the community. These data are likely to support what health promotion effort? A) Education about home safety B) Education about safe storage of chemicals C) Education about workplace health threats D) Education about safe driving

Ans: A Feedback: A large majority of burns occur in the home setting; educational interventions should address this epidemiologic trend.

A nurse on a burn unit is caring for a patient in the acute phase of burn care. While performing an assessment during this phase of burn care, the nurse recognizes that airway obstruction related to upper airway edema may occur up to how long after the burn injury? A) 2 days B) 3 days C) 5 days D) 1 week

Ans: A Feedback: Airway obstruction caused by upper airway edema can take as long as 48 hours to develop. Changes detected by x-ray and arterial blood gases may occur as the effects of resuscitative fluid and the chemical reaction of smoke ingredients with lung tissues become apparent.

A patient is admitted to the burn unit after being transported from a facility 1000 miles away. The patient has burns to the groin area and circumferential burns to both upper thighs. When assessing the patient's legs distal to the wound site, the nurse should be cognizant of the risk of what complication? A) Ischemia B) Referred pain C) Cellulitis D) Venous thromboembolism (VTE)

Ans: A Feedback: As edema increases, pressure on small blood vessels and nerves in the distal extremities causes an obstruction of blood flow and consequent ischemia. This complication is similar to compartment syndrome. Referred pain, cellulitis, and VTE are not noted complications that occur distal to the injury site.

A burn patient is transitioning from the acute phase of the injury to the rehabilitation phase. The patient tells the nurse, ìI can't wait to have surgery to reconstruct my face so I look normal again.î What would be the nurse's best response? A) ìThat's something that you and your doctor will likely talk about after your scars mature.î B) ìThat is something for you to talk to your doctor about because it's not a nursing responsibility.î C) ìI know this is really important to you, but you have to realize that no one can make you look like you used to.î D) ìUnfortunately, it's likely that you will have most of these scars for the rest of your life.î

Ans: A Feedback: Burn reconstruction is a treatment option after all scars have matured and is discussed within the first few years after injury. Even though this is not a nursing responsibility, the nurse should still respond appropriately to the patient's query. It is true that the patient will not realistically look like he or she used to, but this does not instill hope.

A patient has been admitted to a burn intensive care unit with extensive full-thickness burns over 25% of the body. After ensuring cardiopulmonary stability, what would be the nurse's immediate, priority concern when planning this patient's care? A) Fluid status B) Risk of infection C) Nutritional status D) Psychosocial coping

Ans: A Feedback: During the early phase of burn care, the nurse is most concerned with fluid resuscitation, to correct large-volume fluid loss through the damaged skin. Infection control and early nutritional support are important, but fluid resuscitation is an immediate priority. Coping is a higher priority later in the recovery period.

A patient in the emergent/resuscitative phase of a burn injury has had blood work and arterial blood gases drawn. Upon analysis of the patient's laboratory studies, the nurse will expect the results to indicate what? A) Hyperkalemia, hyponatremia, elevated hematocrit, and metabolic acidosis B) Hypokalemia, hypernatremia, decreased hematocrit, and metabolic acidosis C) Hyperkalemia, hypernatremia, decreased hematocrit, and metabolic alkalosis D) Hypokalemia, hyponatremia, elevated hematocrit, and metabolic alkalosis

Ans: A Feedback: Fluid and electrolyte changes in the emergent/resuscitative phase of a burn injury include hyperkalemia related to the release of potassium into the extracellular fluid, hyponatremia from large amounts of sodium lost in trapped edema fluid, hemoconcentration that leads to an increased hematocrit, and loss of bicarbonate ions that results in metabolic acidosis.

A patient has sustained a severe burn injury and is thought to have an impaired intestinal mucosal barrier. Since this patient is considered at an increased risk for infection, what intervention will best assist in avoiding increased intestinal permeability and prevent early endotoxin translocation? A) Early enteral feeding B) Administration of prophylactic antibiotics C) Bowel cleansing procedures D) Administration of stool softeners

Ans: A Feedback: If the intestinal mucosa receives some type of protection against permeability change, infection could be avoided. Early enteral feeding is one step to help avoid this increased intestinal permeability and prevent early endotoxin translocation. Antibiotics are seldom prescribed prophylactically because of the risk of promoting resistant strains of bacteria. A bowel cleansing procedure would not be ordered for this patient. The administration of stool softeners would not assist in avoiding increased intestinal permeability and prevent early endotoxin translocation.

An emergency department nurse has just received a patient with burn injuries brought in by ambulance. The paramedics have started a large-bore IV and covered the burn in cool towels. The burn is estimated as covering 24% of the patient's body. How should the nurse best address the pathophysiologic changes resulting from major burns during the initial burn-shock period? A) Administer IV fluids B) Administer broad-spectrum antibiotics C) Administer IV potassium chloride D) Administer packed red blood cells

Ans: A Feedback: Pathophysiologic changes resulting from major burns during the initial burn-shock period include massive fluid losses. Addressing these losses is a major priority in the initial phase of treatment. Antibiotics and PRBCs are not normally administered. Potassium chloride would exacerbate the patient's hyperkalemia.

An emergency department nurse learns from the paramedics that they are transporting a patient who has suffered injury from a scald from a hot kettle. What variables will the nurse consider when determining the depth of burn? A) The causative agent B) The patient's preinjury health status C) The patient's prognosis for recovery D) The circumstances of the accident

Ans: A Feedback: The following factors are considered in determining the depth of a burn: how the injury occurred, causative agent (such as flame or scalding liquid), temperature of the burning agent, duration of contact with the agent, and thickness of the skin. The patient's preinjury status, circumstances of the accident, and prognosis for recovery are important, but are not considered when determining the depth of the burn.

A patient who is in the acute phase of recovery from a burn injury has yet to experience adequate pain control. What pain management strategy is most likely to meet this patient's needs? A) A patient-controlled analgesia (PCA) system B) Oral opioids supplemented by NSAIDs C) Distraction and relaxation techniques supplemented by NSAIDs D) A combination of benzodiazepines and topical anesthetics

Ans: A Feedback: The goal of treatment is to provide a long-acting analgesic that will provide even coverage for this long-term discomfort. It is helpful to use escalating doses when initiating the medication to reach the level of pain control that is acceptable to the patient. The use of patient-controlled analgesia (PCA) gives control to the patient and achieves this goal. Patients cannot normally achieve adequate pain control without the use of opioids, and parenteral administration is usually required.

A nurse has reported for a shift at a busy burns and plastics unit in a large university hospital. Which patient is most likely to have life-threatening complications? A) A 4-year-old scald victim burned over 24% of the body B) A 27-year-old male burned over 36% of his body in a car accident C) A 39-year-old female patient burned over 18% of her body D) A 60-year-old male burned over 16% of his body in a brush fire

Ans: A Feedback: Young children and the elderly continue to have increased morbidity and mortality when compared to other age groups with similar injuries and present a challenge for burn care. This is an important factor when determining the severity of injury and possible outcome for the patient.

A patient is in the acute phase of a burn injury. One of the nursing diagnoses in the plan of care is Ineffective Coping Related to Trauma of Burn Injury. What interventions appropriately address this diagnosis? Select all that apply. A) Promote truthful communication. B) Avoid asking the patient to make decisions. C) Teach the patient coping strategies. D) Administer benzodiazepines as ordered. E) Provide positive reinforcement.

Ans: A, C, E Feedback: The nurse can assist the patient to develop effective coping strategies by setting specific expectations for behavior, promoting truthful communication to build trust, helping the patient practice appropriate strategies, and giving positive reinforcement when appropriate. The patient may benefit from being able to make decisions regarding his or her care. Benzodiazepines may be needed for short-term management of anxiety, but they are not used to enhance coping.

A patient's burns have required a homograft. During the nurse's most recent assessment, the nurse observes that the graft is newly covered with purulent exudate. What is the nurse's most appropriate response? A) Perform mechanical dÈbridement to remove the exudate and prevent further infection. B) Inform the primary care provider promptly because the graft may need to be removed. C) Perform range of motion exercises to increase perfusion to the graft site and facilitate healing. D) Document this finding as an expected phase of graft healing.

Ans: B Feedback: An infected graft may need to be removed, thus the care provider should be promptly informed. ROM exercises will not resolve this problem and the nurse would not independently perform dÈbridement.

The nurse is preparing the patient for mechanical dÈbridement and informs the patient that this will involve which of the following procedures? A) A spontaneous separation of dead tissue from the viable tissue B) Removal of eschar until the point of pain and bleeding occurs C) Shaving of burned skin layers until bleeding, viable tissue is revealed D) Early closure of the wound

Ans: B Feedback: Mechanical dÈbridementcan be achieved through the use of surgical scissors, scalpels, or forceps to remove the eschar until the point of pain and bleeding occurs. Mechanical dÈbridement can also be accomplished through the use of topical enzymatic dÈbridement agents. The spontaneous separation of dead tissue from the viable tissue is an example of natural dÈbridement. Shaving the burned skin layers and early wound closure are examples of surgical dÈbridement.

A patient who was burned in a workplace accident has completed the acute phase of treatment and the plan of care has been altered to prioritize rehabilitation. What nursing action should be prioritized during this phase of treatment? A) Monitoring fluid and electrolyte imbalances B) Providing education to the patient and family C) Treating infection D) Promoting thermoregulation

Ans: B Feedback: Patient and family education is a priority during rehabilitation. There should be no fluid and electrolyte imbalances in the rehabilitation phase. The presence of impaired thermoregulation or infection would suggest that the patient is still in the acute phase of burn recovery.

A nurse who is taking care of a patient with burns is asked by a family member why the patient is losing so much weight. The patient is currently in the intermediate phase of recovery. What would be the nurse's most appropriate response to the family member? A) ìHe's on a calorie-restricted diet in order to divert energy to wound healing.î B) ìHis body has consumed his fat deposits for fuel because his calorie intake is lower than normal.î C) ìHe actually hasn't lost weight. Instead, there's been a change in the distribution of his body fat.î D) ìHe lost many fluids while he was being treated in the emergency phase of burn care.î

Ans: B Feedback: Patients lose a great deal of weight during recovery from severe burns. Reserve fat deposits are catabolized as a result of hypermetabolism. Patients are not placed on a calorie restriction during recovery and fluid losses would not account for weight loss later in the recovery period. Changes in the overall distribution of body fat do not occur.

A home care nurse is performing a visit to a patient's home to perform wound care following the patient's hospital treatment for severe burns. While interacting with the patient, the nurse should assess for evidence of what complication? A) Psychosis B) Post-traumatic stress disorder C) Delirium D) Vascular dementia

Ans: B Feedback: Post-traumatic stress disorder (PTSD) is the most common psychiatric disorder in burn survivors, with a prevalence that may be as high as 45%. As a result, it is important for the nurse to assess for this complication of burn injuries. Psychosis, delirium, and dementia are not among the noted psychiatric and psychosocial complications of burns.

A nurse who provides care on a burn unit is preparing to apply a patient's ordered topical antibiotic ointment. What action should the nurse perform when administering this medication? A) Apply the new ointment without disturbing the existing layer of ointment. B) Apply the ointment using a sterile tongue depressor. C) Apply a layer of ointment approximately 1/16 inch thick. D) Gently irrigate the wound bed after applying the antibiotic ointment.

Ans: C Feedback: After removing the old ointment from the wound bed, the nurse should apply a layer of ointment 1/16-inch thick using clean gloves. The wound would not be irrigated after application of new ointment.

A triage nurse in the emergency department (ED) receives a phone call from a frantic father who saw his 4-year-old child tip a pot of boiling water onto her chest. The father has called an ambulance. What would the nurse in the ED receiving the call instruct the father to do? A) Cover the burn with ice and secure with a towel. B) Apply butter to the area that is burned. C) Immerse the child in a cool bath. D) Avoid touching the burned area under any circumstances.

Ans: C Feedback: After the flames or heat source have been removed or extinguished, the burned area and adherent clothing are soaked with cool water briefly to cool the wound and halt the burning process. Cool water is the best first-aid measure. Ice and butter are contraindicated. Appropriate first aid necessitates touching the burn.

A patient experienced a 33% TBSA burn 72 hours ago. The nurse observes that the patient's hourly urine output has been steadily increasing over the past 24 hours. How should the nurse best respond to this finding? A) Obtain an order to reduce the rate of the patient's IV fluid infusion. B) Report the patient's early signs of acute kidney injury (AKI). C) Recognize that the patient is experiencing an expected onset of diuresis. D) Administer sodium chloride as ordered to compensate for this fluid loss.

Ans: C Feedback: As capillaries regain integrity, 48 or more hours after the burn, fluid moves from the interstitial to the intravascular compartment and diuresis begins. This is an expected development and does not require a reduction in the IV infusion rate or the administration of NaCl. Diuresis is not suggestive of AKI.

A patient with severe burns is admitted to the intensive care unit to stabilize and begin fluid resuscitation before transport to the burn center. The nurse should monitor the patient closely for what signs of the onset of burn shock? A) Confusion B) High fever C) Decreased blood pressure D) Sudden agitation

Ans: C Feedback: As fluid loss continues and vascular volume decreases, cardiac output continues to decrease and the blood pressure drops, marking the onset of burn shock. Shock and the accompanying hemodynamic changes are not normally accompanied by confusion, fever, or agitation.

A patient with a partial-thickness burn injury had Biobrane applied 2 weeks ago. The nurse notices that the Biobrane is separating from the burn wound. What is the nurse's most appropriate intervention? A) Reinforce the Biobrane dressing with another piece of Biobrane. B) Remove the Biobrane dressing and apply a new dressing. C) Trim away the separated Biobrane. D) Notify the physician for further emergency-related orders.

Ans: C Feedback: As the Biobrane gradually separates, it is trimmed, leaving a healed wound. When the Biobrane dressing adheres to the wound, the wound remains stable and the Biobrane can remain in place for 3 to 4 weeks. There is no need to reinforce the Biobrane nor to remove it and apply a new dressing. There is not likely any need to notify the physician for further orders.

A patient has experienced burns to his upper thighs and knees. Following the application of new wound dressings, the nurse should perform what nursing action? A) Instruct the patient to keep the wound site in a dependent position. B) Administer PRN analgesia as ordered. C) Assess the patient's peripheral pulses distal to the dressing. D) Assist with passive range of motion exercises to ìsetî the new dressing.

Ans: C Feedback: Dressings can impede circulation if they are wrapped too tightly. The peripheral pulses must be checked frequently and burned extremities elevated. Dependent positioning does not need to be maintained. PRN analgesics should be administered prior to the dressing change. ROM exercises do not normally follow a dressing change.

The current phase of a patient's treatment for a burn injury prioritizes wound care, nutritional support, and prevention of complications such as infection. Based on these care priorities, the patient is in what phase of burn care? A) Emergent B) Immediate resuscitative C) Acute D) Rehabilitation

Ans: C Feedback: The acute or intermediate phase of burn care follows the emergent/resuscitative phase and begins 48 to 72 hours after the burn injury. During this phase, attention is directed toward continued assessment and maintenance of respiratory and circulatory status, fluid and electrolyte balance, and gastrointestinal function. Infection prevention, burn wound care (i.e., wound cleaning, topical antibacterial therapy, wound dressing, dressing changes, wound dÈbridement, and wound grafting), pain management, and nutritional support are priorities at this stage. Priorities during the emergent or immediate resuscitative phase include first aid, prevention of shock and respiratory distress, detection and treatment of concomitant injuries, and initial wound assessment and care. The priorities during the rehabilitation phase include prevention of scars and contractures, rehabilitation, functional and cosmetic reconstruction, and psychosocial counseling.

A nurse is developing a care plan for a patient with a partial-thickness burn, and determines that an appropriate goal is to maintain position of joints in alignment. What is the best rationale for this intervention? A) To prevent neuropathies B) To prevent wound breakdown C) To prevent contractures D) To prevent heterotopic ossification

Ans: C Feedback: To prevent the complication of contractures, the nurse will establish a goal to maintain position of joints in alignment. Gentle range of motion exercises and a consult to PT and OT for exercises and positioning recommendations are also appropriate interventions for the prevention of contractures. Joint alignment is not maintained specifically for preventing neuropathy, wound breakdown, or heterotopic ossification.

A patient is brought to the emergency department from the site of a chemical fire, where he suffered a burn that involves the epidermis, dermis, and the muscle and bone of the right arm. On inspection, the skin appears charred. Based on these assessment findings, what is the depth of the burn on the patient's arm? A) Superficial partial-thickness B) Deep partial-thickness C) Full partial-thickness D) Full-thickness

Ans: D Feedback: A full-thickness burn involves total destruction of the epidermis and dermis and, in some cases, underlying tissue as well. Wound color ranges widely from white to red, brown, or black. The burned area is painless because the nerve fibers are destroyed. The wound can appear leathery; hair follicles and sweat glands are destroyed. Edema may also be present. Superficial partial-thickness burns involve the epidermis and possibly a portion of the dermis; the patient will experience pain that is soothed by cooling. Deep partial-thickness burns involve the epidermis, upper dermis, and portion of the deeper dermis; the patient will complain of pain and sensitivity to cold air. Full partial thickness is not a depth of burn.

The nurse caring for a patient who is recovering from full-thickness burns is aware of the patient's risk for contracture and hypertrophic scarring. How can the nurse best mitigate this risk? A) Apply skin emollients as ordered after granulation has occurred. B) Keep injured areas immobilized whenever possible to promote healing. C) Administer oral or IV corticosteroids as ordered. D) Encourage physical activity and range of motion exercises.

Ans: D Feedback: Exercise and the promotion of mobility can reduce the risk of contracture and hypertrophic scarring. Skin emollients are not normally used in the treatment of burns, and these do not prevent scarring. Steroids are not used to reduce scarring, as they also slow the healing process.

A patient's burns are estimated at 36% of total body surface area; fluid resuscitation has been ordered in the emergency department. After establishing intravenous access, the nurse should anticipate the administration of what fluid? A) 0.45% NaCl with 20 mEq/L KCl B) 0.45% NaCl with 40 mEq/L KCl C) Normal saline D) Lactated Ringer's

Ans: D Feedback: Fluid resuscitation with lactated Ringers (LR) should be initiated using the American Burn Association's (ABA) fluid resuscitation formulas. LR is the crystalloid of choice because its composition and osmolality most closely resemble plasma and because use of normal saline is associated with hyperchloremic acidosis. Potassium chloride solutions would exacerbate the hyperkalemia that occurs following burn injuries.

An occupational health nurse is called to the floor of a factory where a worker has sustained a flash burn to the right arm. The nurse arrives and the flames have been extinguished. The next step is to ìcool the burn.î How should the nurse cool the burn? A) Apply ice to the site of the burn for 5 to 10 minutes. B) Wrap the patient's affected extremity in ice until help arrives. C) Apply an oil-based substance or butter to the burned area until help arrives. D) Wrap cool towels around the affected extremity intermittently.

Ans: D Feedback: Once the burn has been sustained, the application of cool water is the best first-aid measure. Soaking the burn area intermittently in cool water or applying cool towels gives immediate and striking relief from pain, and limits local tissue edema and damage. However, never apply ice directly to the burn, never wrap the person in ice, and never use cold soaks or dressings for longer than several minutes; such procedures may worsen the tissue damage and lead to hypothermia in people with large burns. Butter is contraindicated.

A nurse is caring for a patient who has sustained a deep partial-thickness burn injury. In prioritizing the nursing diagnoses for the plan of care, the nurse will give the highest priority to what nursing diagnosis? A) Activity Intolerance B) Anxiety C) Ineffective Coping D) Acute Pain

Ans: D Feedback: Pain is inevitable during recovery from any burn injury. Pain in the burn patient has been described as one of the most severe causes of acute pain. Management of the often-severe pain is one of the most difficult challenges facing the burn team. While the other nursing diagnoses listed are valid, the presence of pain may contribute to these diagnoses. Management of the patient's pain is the priority, as it may have a direct correlation to the other listed nursing diagnoses.

A nurse is caring for a patient with burns who is in the later stages of the acute phase of recovery. The plan of nursing care should include which of the following nursing actions? A) Maintenance of bed rest to aid healing B) Choosing appropriate splints and functional devices C) Administration of beta adrenergic blockers D) Prevention of venous thromboembolism

Ans: D Feedback: Prevention of deep vein thrombosis (DVT) is an important factor in care. Early mobilization of the patient is important. The nurse monitors the splints and functional devices, but these are selected by occupational and physical therapists. The hemodynamic changes accompanying burns do not normally require the use of beta blockers.

A nurse is performing a home visit to a patient who is recovering following a long course of inpatient treatment for burn injuries. When performing this home visit, the nurse should do which of the following? A) Assess the patient for signs of electrolyte imbalances. B) Administer fluids as ordered. C) Assess the risk for injury recurrence. D) Assess the patient's psychosocial state.

Ans: D Feedback: Recovery from burns can be psychologically challenging; the nurse's assessments must address this reality. Fluid and electrolyte imbalances are infrequent during the rehabilitation phase of recovery. Burns are not typically a health problem that tends to recur; the experience of being burned tends to foster vigilance.

An emergency department nurse has just admitted a patient with a burn. What characteristic of the burn will primarily determine whether the patient experiences a systemic response to this injury? A) The length of time since the burn B) The location of burned skin surfaces C) The source of the burn D) The total body surface area (TBSA) affected by the burn

Ans: D Feedback: Systemic effects are a result of several variables. However, TBSA and wound severity are considered the major factors that affect the presence or absence of systemic effects.

A patient is brought to the ED by paramedics, who report that the patient has partial-thickness burns on the chest and legs. The patient has also suffered smoke inhalation. What is the priority in the care of a patient who has been burned and suffered smoke inhalation? A) Pain B) Fluid balance C) Anxiety and fear D) Airway management

Ans: D Feedback: Systemic threats from a burn are the greatest threat to life. The ABCs of all trauma care apply during the early postburn period. While all options should be addressed, pain, fluid balance, and anxiety and fear do not take precedence over airway management.

dressings for wound debridement (2)

Aquacel (Ag) & Mepliex (Ag) -Ag= silver (has an anitimicrobial agent and prevents infection) -applied directly to the wound and stays in place for up to 1 week -helps the wound heal faster with less scaring -can only use a silver dressing for up to 3 weeks -if not healed after 3 weeks may require a skin graft

Rule of 9's-

Assesses the percentage of burn and is used to help guide treatment decisions including fluid resuscitation and becomes part of the guidelines to determine transfer to a burn unit

Second degress burn; Partial thickenss burn

Blisters, painful, swelling, intact hair, cap refill is present. Example: scalds and flash burns. Effects epidermis, upper dermis, and portions of the deeper dermis.

Rehabilitation Phase Nursing and Collaborative Management

Both patient and family actively learn how to care for healing wounds An emollient water-based cream should be used Cosmetic surgery is often needed after major burns

If patient has a burn on the face and the torso, what is the priority assesment?

Breathing

4th degree

Burn that extends into fascia and bone

When does a patient need fluid resuscitation?

Burns greater than 20% of total body surface area

The patient sustained a full-thickness burn encompassing the entire right arm. What is the best indicator an escharotomy achieved its desired effect? A. Patient rates the pain at less than 4. B. Blood pressure remains above 120/80 mm Hg. C. Right fingers blanch with a 2-second refill. D. Patient maintains full range of motion for the right arm.

C. Circulation to the extremities can be severely impaired by deep circumferential burns and subsequent edema that impairs the blood supply. An escharotomy (electrocautery incision through the full-thickness eschar) is performed to restore circulation. Normal refill is less than 2 seconds.

What would be an appropriate intervention for a nursing diagnosis of "Impaired skin integrity R/T open burn wounds"

Clean clients wounds, body, and hair daily to reduce bacterial colonization

What to do with patient when transferring?

Clean, dry sheets to cover the patient and keep patient warm and dry

Why are auto-grafts preferred when possible?

Clients own skin, less likely to reject

How do you measure emergency burns?

Compare clients palm with the size of the burn wound. The palm is approx. 1% of a person's total body surface area.

Electrical Burns (Cont'd)

Current that passes through vital organs will produce more life-threatening sequelae than current passing through other tissue Put a patient at risk for dysrhythmias, severe metabolic acidosis, and myoglobinuria Severity of injury can be difficult to assess because most damage is beneath the skin - iceberg effect Electrical sparks may ignite the patient's clothing, causing a combination of thermal and electrical injury

Multiple patients arrive in the emergency department from a house fire. Which patient is a priority? A. Patient with erythremic, dry burns over the arms and a history of taking prednisone B. Patient with moist blisters over the chest and who reports pain as 10 C. Patient with dry, black skin on one hand and a history of diabetes mellitus D. Patient with multiple reddened skin areas on the chest and with high-pitched respiratory sounds

D. Airway injury is a priority, and stridor results from a narrowing of the airway caused by edema. A history of prednisone use or diabetes is a concern for long-term infection risk, but the airway is always first.

what do you do for larger burned areas

DO NOT flush with cool water!! -already losing heat and by flushing with cool water you put the patient at risk for *hypothermia*

Acute Phase Pathophysiology

Diuresis from fluid mobilization occurs, and the patient is no longer grossly edematous Bowel sounds return Healing begins when WBCs have surrounded the burn wound and phagocytosis occurs

Emergent Phase Nursing and Collaborative Management (Cont'd)

Drug therapy Analgesics and sedatives Antimicrobial agents Tetanus immunization Given routinely to all burn patients

Prehospital Care

Electrical injuries Remove patient from contact with source Chemical injuries Brush solid particles off the skin Water lavage Small thermal burns Cover with clean, cool, tap water- dampened towel Large thermal burns Airway, breathing, and circulation Do not immerse in cool water or pack with ice Wrap in clean, dry sheet or blanket Remove burned clothing

Emergent Phase

Emergent phase is the period of time required to resolve the immediate problems resulting from injury Usually lasts 24 to 48 hours The phase begins with fluid loss and edema formation and continues until fluid mobilization and diuresis begin

The nurse is concerned with a burn patients nutritional status. Which intervention is best?

Encourage patients family to bring favorite foods

Acute Phase Complications (Cont'd)

Endocrine system ↑ Blood glucose levels ↑ Insulin production Hyperglycemia

The patient had a burn to the left arm and had a skin graft, how will you tell if it was succesful?

Feel for pulses bilaterally, or at least in affected extremity

Emergent Phase Pathophysiology

Fluid and electrolyte shifts Greatest threat is hypovolemic shock caused by a massive shift of fluids out of blood vessels as a result of increased capillary permeability Normal insensible loss: 30 to 50 ml/hr Severely burned patient: 200 to 400 ml/hr

Emergent Phase Pathophysiology

Fluid and electrolyte shifts (cont'd) Colloidal osmotic pressure decreases, resulting in more fluid shifting out of the vascular space into the interstitial spaces The net result of the fluid shift is intravascular volume depletion Edema ↓ Blood pressure ↑ Pulse Fluid and electrolyte shifts (cont'd) RBCs are hemolyzed by a circulating factor released at the time of the burn Thrombosis Elevated hematocrit

Emergent Phase Pathophysiology

Fluid and electrolyte shifts (cont'd) Na+ shifts to the interstitial spaces and remains until edema formation ceases K+ shift develops because injured cells and hemolyzed RBCs release K+ into extracellular spaces

Emergent Phase Nursing and Collaborative Management

Fluid therapy One or two large-bore IV lines Type of fluid replacement based on size/depth of burn, age, and individual considerations Parkland (Baxter) formula for fluid replacement Colloidal solutions

What is the priority action for a chemical burn?

Flush with copius amounts of water

How do you assess electrical burns?

Follow an internal path from entrance to exit of arch

Xenograft

Graft coming from an animal, usually a pig

Nursing intervention for patient who has full thickness and deep partial thickness burns to 50% of body?

Hand hygiene is #1, sterile technique, gown and mask, Invasive lines changed daily, and antibiotics

During early phase of burn care, assess the client for?

Hyperkalemia - can lead to heart irregularities

never use what with burns

ICE

what should you avoid with burns?

IM or subQ injections (reabsorption is delayed and the soft tissue is unreliable)

Which meds to use in acute phase of burn to decrease perception of pain?

IV Opioids

What would you anticipate for a graft on the lower extremities?

Immobilization of affected leg (at least 3-4 days)

If the patient has a skin graft to the right knee, how soon can they get up after surgery?

Immobilize right leg up to 7 days, at least 3

Emergent Phase Pathophysiology (Cont'd)

Immunologic changes Burn injury causes widespread impairment of the immune system The skin barrier is destroyed Decreased circulating levels of immunoglobulins WBC changes

Rehabilitation Phase Pathophysiologic Changes (Cont'd)

In approximately 4 to 6 weeks the area becomes raised and hyperemic Mature healing is reached in 6 months to 2 years Skin never completely regains its original color

What to do if patient isnt putting out enough urine?

Increase LR rate

Acute Phase Complications

Infection Partial-thickness burns can become full-thickness wounds in the presence of infection Wound infection may progress to transient bacteremia Patient may develop sepsis

Emergent Phase Pathophysiology

Inflammation and healing Neutrophils and monocytes accumulate at the site of injury Fibroblasts and collagen fibrils begin wound repair within the first 6 to 12 hours of injury

Smoke Inhalation Injuries (Cont'd)

Inhalation injury above the glottis Thermally produced Hot air, steam, or smoke Mucosal burns of oropharynx and larynx Mechanical obstruction can occur quickly True medical emergency Types of Burn Injury

Smoke Inhalation Injuries (Cont'd)

Inhalation injury below the glottis Injury is related to the length of exposure to smoke or toxic fumes Pulmonary edema may not appear until 12 to 24 hours after the burn Then may manifest as acute respiratory distress syndrome Types of Burn Injury

When do you use supplemented O2?

Inhalation injury, flame burns, burns to face or chest, or greater than 20% of BSA

What is the IV fluid of choice?

LR

Location of Burn

Location of the burn is related to the severity of the injury Circumferential burns of the extremities can cause circulatory compromise Patients may also develop compartment syndrome Classification of Burn Injury

Signs of a full thickness burn?

Loss of sensation, may not feel pain

Goal of fluid resuscitation?

Maintaining vital organ function by providing adequate intravascular volumes

What do you monitor with Sulfamylon, a sulfa antibiotic cream?

Met. acidosis

What are pressure garments used for?

Minimize scaring and the over growth of skin

Acute Phase Complications (Cont'd)

Musculoskeletal system Decreased ROM Contractures Gastrointestinal system Paralytic ileus Curling's ulcer

Acute Phase Pathophysiology (Cont'd)

Necrotic tissue begins to slough Formation of granulation tissue A partial-thickness burn wound will heal from the edges Full-thickness burns must be covered by skin grafts

Emotional Needs of the Patient and Family

Need to understand the importance of reestablishing the pt's independence Needto participate as team members Early psychiatric intervention A common emotional response is anger/regression Issue of sexuality must be met with honesty

Acute Phase Complications (Cont'd)

Neurologic system Usually no problems unless severe hypoxia occurs Disorientation Combative Hallucinations Delirium ICU psychosis syndrome Use of analgesics and antianxiety drugs

When estimating BSA, do you include first degree burns?

No

Is it okay to give burn patients IM shots?

No, it won't be absorbed due to vascular compromise and disturbed skin integrity

Gerontologic Considerations

Normal aging puts the patient at risk for injury because of: Unsteady gait Failing eyesight Diminished hearing Preexisting medical conditions Wounds take longer to heal

Emergent Phase Nursing and Collaborative Management (Cont'd)

Nutritional therapy Fluid replacement takes priority over nutritional needs Early and aggressive nutritional support within hours of burn injury Decrease mortality and complications Optimize wound healing Minimize negative effects

Acute Phase Nursing and Collaborative Management (Cont'd)

Nutritional therapy Meeting daily caloric requirements is critical Caloric needs should be calculated by dietitian High-protein, high-carbohydrate foods Diet supplements Patients should be weighed regularly

Emergent Phase Nursing and Collaborative Management

Nutritional therapy (cont'd) Hypermetabolic state Resting metabolic expenditure may be increased by 50% to 100% above normal Core temperature is elevated Caloric needs are about 5000 kcal/day Early, continuous enteral feeding promotes optimal conditions for wound healing Supplemental vitamins and iron may be given Acute Phase: The acute phase begins with the mobilization of extracellular fluid and subsequent diuresis The acute phase is concluded when the burned area is completely covered by skin grafts or when the wounds are healed

Burns

Occur when there is injury to the tissues of the body caused by heat, chemicals, electrical current, or radiation Types of Burn Injury: Thermal burns - fire most common Chemical burns - destruction from acids, alkalis, and organic compounds Smoke inhalation - inhalation of hot air or noxious chemicals and major predictor of prognosis Electrical burns Cold thermal injuries Types of Burn Injury

Patient Risk Factors

Older adults heal more slowly than young adults Preexisting cardiovascular, respiratory, and renal disease cause for poorer prognosis Diabetes mellitus contributes to poor healing and gangrene Classification of Burn Injury

Patient with deep partial thickness and full thickness burns. Which data would warrant notifying HCP?

Output of 50ml/2hr

If your patient has an electrical burn, what do you need to worry about their urine?

Output of 75-100ml; keep more diluted

hyperkalemia from the burn causes an EKG to look like what?

PEAKED T WAVES

Acute Phase Nursing and Collaborative Management (Cont'd)

Pain management Patients experience two kinds of pain Continuous background pain Treatment-induced pain Several drugs in combination Morphine with haloperidol Treatment-induced pain managed with potent, short-acting analgesic

Refferal criteria to burn unit?

Partial thickness burns greater than 10% BSA, Burns on face, hands, feet, genitalia, perineum, and major joints, any third degree burns, electric burns including lightning, chemical burns, inhalation injuries, pre existing medical disorders, burns and concomitant trauma, hospitals w/o equipment or personnel to care for children, and burns in patients who will require special social, emotional and or long term rehab

Acute Phase Clinical Manifestations

Partial-thickness wounds form eschar Once eschar is removed, re-epithelialization begins Full-thickness wounds require debridement

Acute Phase Nursing and Collaborative Management (Cont'd)

Physical and occupational therapy Good time for exercise is during wound cleaning Passive and active ROM Splints should be custom fitted

Patient Risk Factors (Cont'd)

Physical debilitation renders patient less able to recover Alcoholism Drug abuse Malnutrition Concurrent fractures, head injuries, or other trauma also lead to poor prognosis

Patient with hot grease burn to right hand calls and asks what to do?

Place hand in cool water. Gives relief and limits local tissue edema and damage

Acute Phase Laboratory Values (Cont'd)

Potassium Hyperkalemia noted if patient has Renal failure Adrenocortical insufficiency Massive deep muscle injury Hyperkalemia can cause Cardiac dysrhythmias and ventricular failure Muscle weakness ECG changes

Acute Phase Laboratory Values

Potassium (cont'd) Hypokalemia can be caused by Lengthy hydrotherapy Vomiting, diarrhea Prolonged gastrointestinal suction and IV therapy without supplementation

Phases of Burn Management

Prehospital care Emergent (resuscitative) Acute (wound healing) Rehabilitative (restorative)

What do you consider with chemical burns?

Protect yourself from further burns in caring for this client

Acute Phase Nursing and Collaborative Management (Cont'd)

Psychosocial care Social worker Nursing staff

Full thickness burn to 65% if body including chest. After establishing a patent airway, what is the priority?

Replace fluids and electrolytes to prevent irreversible shock

Emergent Phase Complications

Respiratory system Upper respiratory tract injury Causes edema formation Mechanical airway obstruction and asphyxia Pneumonia Pulmonary edema Inhalation injury Direct insult at the alveolar level Interstitial edema Patient may not exhibit signs during first 24 hours

Electrical Burns

Result from coagulation necrosis caused by intense heat generated from an electrical current May result from direct damage to nerves and vessels causing tissue anoxia and death Types of Burn Injury

Classification of Burn Injury

Severity of injury is determined by Depth of burn Extent of burn Location of burn Patient risk factors Classification of Burn Injury

Emergent Phase Clinical Manifestations

Shock from pain and hypovolemia Blisters Adynamic ileus Shivering Altered mental status

Nursing considerations when applying antimicrobial agents to burns?

Silver Sulfadiazine (Silvadene)- only apply to clean skin

Rehabilitation Phase Complications

Skin and joint contractures Most common complications during rehab phase Hypertrophic scarring

What is natural debridement?

Sloughs on its own

Acute Phase Laboratory Values

Sodium Hyponatremia can occur if hydrotherapy is longer than 20 to 30 minutes or from excessive GI drainage Hypernatremia May develop after successful fluid replacement Improper tube feedings

Depth of Burn (Cont'd)

Superficial partial-thickness burn Involves the epidermis Deep partial-thickness burn Involves the dermis Full-thickness burn Involves fat, muscle, bone Classification of Burn Injury

Patient is discharged after being in the burn unit for 6 weeks. Which strategies should the nurse identify to promote the client's mental health?

Tell client to remember that changes in lifestyle take time

Depth of Burn

The ABA advocates categorizing the burn according to depth of skin destruction, not 1st , 2nd or 3rd degree: Partial-thickness burn Full-thickness burn Classification of Burn Injury

Rehabilitation Phase Pathophysiologic Changes

The burn wound heals either by primary intention or by grafting Layers of epithelialization begin rebuilding the tissue structure Collagen fibers add strength to weakened areas

Who is most likely to require a trach?

Thermal burns to head, face, and airway resulting in hypoxia

When is an NG tube suggested?

Ventilated patients or if BSA is greater than 20%

Acute Phase Nursing and Collaborative Management

Wound care Daily observation Assessment Cleansing Debridement Dressing reapplication

Emergent Phase Nursing and Collaborative Management

Wound care Cleansing Can be done in a hydrotherapy tub, shower, or bed Debridement May need to be done in the OR Loose necrotic skin is removed Infection is the most serious threat to further tissue injury Source of infection is the patient's own flora

Emergent Phase Nursing and Collaborative Management

Wound care Open method Burn is covered with a topical antibiotic with no dressing over the wound Multiple dressing changes Sterilized gauze dressings are laid over a topical antibiotic Dressings may be changed from 1 to 3 times every 12 hours to once every 3 days

Acute Phase Nursing and Collaborative Management

Wound care (cont'd) Appropriate coverage of the graft: Fine-mesh gauze next to the graft followed by middle and outer dressings Sheet skin grafts must be kept free of blebs

The nurse is planning care for a patient with partial- and full-thickness skin destruction related to burn injury of the lower extremities. Which of the following interventions would the nurse expect to include in this patient's care (select all that apply)? A.Escharotomy B.Administration of diuretics C.IV and oral pain medications D.Daily cleansing and debridement E.Application of topical antimicrobial agent

a, c, d, e. An escharotomy (a scalpel incision through full-thickness eschar) is frequently required to restore circulation to compromised extremities. Daily cleansing and debridement as well as application of an antimicrobial ointment are expected interventions used to minimize infection and enhance wound healing. With full-thickness burns, myoglobin and hemoglobin released into the bloodstream can occlude renal tubules. Adequate fluid replacement is used to prevent this occlusion. Pain control is essential in the care of a patient with a burn injury

A therapeutic measure used to prevent hypertrophic scarring during the rehabilitative phase of burn recovery is: a. applying pressure garments. b. repositioning the pt every 2 hours c. performing active ROM at least every 4 hours d. massaging the new tissue with water based moisturizers

a. applying pressure garments. Pressure can help keep a scar flat and reduce hypertrophic scarring. Gentle pressure can be maintained on the healed burn with custom-fitted pressure garments.

A child was admitted to the ED with a thermal burn to the right arm and leg. Which assessment by the nurse requires immediate action? a. coughing and wheezing b. bright red skin with small blister on the burn sites c. thirst d. singed hair

a. coughing and wheezing may indicate that the child has inhaled smoke or toxic fumes. Maintaining airway patency is the highest nursing priority in this situation.

The nurse is caring for a client who suffered an inhalation injury from a wood stove. The carbon monoxide blood report reveals a level of 12%. Based on this level, the nurse would anticipate which of the following signs in the client? a. coma b. flushing c. dizziness d. tachycardia

b. 11-20% - signs include flushing, headache, decreased visual acuity, decreased cerebral functioning, and slight breathlessness. 21-40% - signs include nausea, vomiting, dizziness, tinnitus, vertigo, confusion, drowsiness, pale to reddish-purple skin, tachycardia; levels of 41-60% result in seizure and come and levels higher than 60% result in death

To maintain a positive nitrogen balance in a major burn, the patient must a. increase normal caloric intake by about 3 times b. eat a high-protein, low-fat, high-carbohydrate diet. c. eat at least 1500 calories per day in small, frequent meals. d. eat rice and whole wheat for the chemical effect on nitrogen balance

b. eat a high-protein, low-fat, high-carbohydrate diet. The patient should be encouraged to eat high-protein, high-carbohydrate foods to meet increased caloric needs. Massive catabolism can occur and is characterized by protein breakdown and increased gluconeogenesis. Failure to supply adequate calories and protein leads to malnutrition and delayed healing.

The nurse plans care for a male pt who suffered thermal burns to the entire posterior aspect of his body when he fell on an outdoor grill. Which pt need is likely to be the primary problem of this pt in the emergent phase? a. maintain tissue oxygenation b. halt progression of the burn c. maintain intravascular volume d. prevent invasion of pathogens

b. the first priority is halting the severity of the burn, to limit the depth of the burn and quick action must be a priority.

Pain management for the burn patient is most effective when a. opoids are administered on a set schedule around the clock b. the patient has as much control over the management of the pain as possible. c. there is flexibility to administer opioids withing a dosage and frequency range d. painful dressing changes are delayed until the pt's pain is totally relieved.

b. the patient has as much control over the management of the pain as possible. The more control the patient has in managing the pain, the more successful the chosen strategies. Active patient participation has been found to be effective for some patients in anticipating and coping with treatment-induced pain.

A patient is to undergo skin grafting with the use of cultured epithelial autografts full-thickness burns. The nurse explains to the patient that this treatment involves a) Shaving a split-thickness layer of the patient's skin to cover the burn wound. b) Using epidermal growth factor to cultivate cadaver skin for temporary wound coverage. c) Growing small specimens of the patient's skin into sheets to use as permanent skin coverage. d) Exposing animal skin to growth factors to decrease antigenicity so it can be used for permanent wound coverage.

c Rationale: Cultured epithelial autograft (CEA) is a method of obtaining permanent skin from a person with limited available skin for harvesting. CEA is grown from biopsy specimens obtained from the patient's own unburned skin.

A client with burn injury asks the nurse what the term full thickness means. The nurse should respond that burns classified as full thickness involve tissue destruction down to which level? a. epidermis b. dermis c. subcutaneous tissue d. internal organs

c. A full thickness burn involves all skin layers, including the epidermis and dermis, and may extend into the subcutaneous tissue and fat.

A nurse is caring for a patient with second- and third-degree burns to 50% of the body. The nurse prepares fluid resuscitation based on knowledge of the Parkland (Baxter) formula that includes which of the following recommendations? A. The total 24-hour fluid requirement should be administered in the first 8 hours. B. One half of the total 24-hour fluid requirement should be administered in the first 8 hours. C. One third of the total 24-hour fluid requirement should be administered in the first 4 hours. D. One half of the total 24-hour fluid requirement should be administered in the first 4 hours.

c. Fluid resuscitation with the Parkland (Baxter) formula recommends that one half of the total fluid requirement should be administered in the first 8 hours, one quarter of total fluid requirement should be administered in the second 8 hours, and one quarter of total fluid requirement should be administered in the third 8 hours.

The nurse is caring for a client following an autograft and grafting to a burn would on the right knee. Which of the following would the nurse anticipate to be prescribed for the client? a. out of bed b. brp c. Immobilization of the affected leg d. placing the affected leg in a dependent position

c. autografts placed over joints or on the lower extremities after surgery often are elevated and immobilized for 3-7 days. this period allows the autograft time to adhere to the wound bed.

What is the best method for preventing hypovolemic shock in a client admitted with severe burns? a. administering dopamine b. applying medical antishock trousers c. infusing i.v. fluids d. infusing fresh frozen plasma

c. during the early postburn period, large amounts of plasma fluid extravasates into interstitial spaces. Restoring the fluid loss is necessary to prevent hypovolemic shock; this is best accomplished with crystalloid and colloid solutions.

Fluid and electrolyte shifts that occur during the early emergent phase include a. adherence of albumin to vascular walls b. movement of potassium into the vascular space c. sequestering of sodium and water in the interstitial fluid. d. hemolysis of red blood cells from large volumes of rapidly administered fluid.

c. sequestering of sodium and water in the interstitial fluid.

A female pt in the acute phase of burn care has electrical burns on the left side of her body, type 2 diabetes mellitus, and a serum glucose of 485 mg/dL. What is the nurse's priority for preventing a life threatening complication of hyperglycemia for the burn patient? a. replace the blood lost b. maintain a neutral pH c. Maintain fluid balance d. Replace serum potassium

c. this pt is most likely experiencing hyperglycemic hyperosmolar nonketotic syndrome (HHNKS) which dehydrates a patient rapidly. This increases the pt's risk for hypovolemia and hypotension.

When monitoring initial fluid replacement for the patient with 40% TBSA deep partial-thickness and full-thickness burns, which finding is of most concern to the nurse? a) Serum K+ of 4.5 mEq/L b) Urine output of 35 mL/hr c) Decreased bowel sounds d) Blood pressure of 86/72 mm Hg

d Rationale: Adequacy of fluid replacement is assessed by urine output and cardiac parameters. Urine output should be 0.5 to 1 mL/kg/hr. Mean arterial pressure should be >65 mm Hg, systolic BP >90 mm Hg, and heart rate <120 beats/min. A blood pressure of 86/72 indicates inadequate fluid replacement. However, the MAP is calculated at 77 mm Hg.

The nurse plans emergent care for four male pt's who have burns covering between 40-50% of the total body surface area. Rank these patients according to their risk for an inhalation injury beginning with the pt who has the highest risk. a. has posterior chemical burns from an exhibit at a parking lot b. has osteoporosis and electrical burns of the lower extremities c. has thermal burns of the right side and is a volunteer fireman d. has chronic bronchitis and thermal burns around the abdomen

d, c, a, b.

The client arrives at the emergency department following a burn injury that occurred in the basement at home and an inhalation injury is suspected. Which of the following would the nurse anticipate to be prescribed for the client? a. 100% oxygen via an aerosol mask b. Oxygen via nasal cannula at 15L/min c. Oxygen via nasal cannula at 10L/min d. 100% oxygen via a tight fitting, non rebreather face mask

d.

The nurse is caring for a patient with partial- and full-thickness burns to 65% of the body. When planning nutritional interventions for this patient, the nurse should implement which of the following dietary choices? A. Full liquids only B. Whatever the patient requests C. High-protein and low-sodium foods D. High-calorie and high-protein foods

d. A hypermetabolic state occurs proportional to the size of the burn area. Massive catabolism can occur and is characterized by protein breakdown and increases gluconeogenesis. Caloric needs are often in the 5000-kcal range. Failure to supply adequate calories and protein leads to malnutrition and delayed healing.

A patient is admitted to the burn center with burns over his head, neck, chest, back, and left arm and hand after an explosion and fire in his garage. On admission to the unit, you auscultate wheezes throughout the lung fields. On reassessment, you notice that the wheezes are gone and the breath sounds are greatly diminished. Which of the following actions is the most appropriate next step? a. place the pt in high fowler's position b. encourage the pt to cough and auscultate the lungs again c. document the results and continue to monitor the pt's progress d. anticipate the need for endotracheal intubation and notify the physician

d. Anticipate the need for endotracheal intubation, and notify the physician. Inhalation injury results in exposure of respiratory tract to intense heat or flames with inhalation of noxious chemicals, smoke, or carbon monoxide. You should anticipate the need for intubation.

A patient has been treated for second- and third-degree burns over 30% of his body and is now ready for discharge. You provide discharge instructions related to wound care. Which statement indicates that the patient understands the instructions? a. I can expect occasional periods of low grade fever and can take Tylenol every 4 hours b. I must wear my jobst elastic garment all day and can only remove it when I'm going to bed. c. I will need to take sponge baths at home to avoid exposing the wounds to unsterile bath water. d. If any healed areas break open, I should cover them with a sterile dressing and then immediately report it."

d. If any healed areas break open, I should cover them with a sterile dressing and then immediately report it."

The nurse is caring for a client who sustained superficial partial thickness burns on the anterior lower legs and anterior thorax. Which of the following does the nurse expect to note during the resuscitation/emergent phase of the burn injury? a. decreased heart rate b. increased urinary output c. increased blood pressure d. elevated hematocrit levels

d. during the resuscitation/emergent phase, the hematocrit level increases to above normal because of hemoconcentration from the large fluid shift.

A patient has 25% TBSA burned from a car fire. His wounds have been debrided and covered with a silver-impregnated dressing. Your priority intervention for wound care is to: a. reapply a new dressing without disturbing the wound bed b. observe the wound for signs of infection during dressing changes c. apply cool compresses for pain relief in between dressing changes d. wash the wound aggressively with soap and water three times each day.

d. wash the wound aggressively with soap and water three times each day. Infection is the most serious threat for further tissue injury and possible sepsis.

why do you need more urinary output with electrical burns?

electrical burns can be worse on the inside so puts the patient at a higher risk for renal failure

First degree burn; Superficial partial thickness burn

example: sunburn. Effects epidermis and skin is reddened and blanches with pressure and is painful

partial thickness burns

first and second degree

what should you do with chemical burns

flush off with water -if from power you need to brush it off then flush

if the burn is <10% TBSA what should you do?

flush the site with cool water -never use iced or ice water

why is wound care to the face and eyes important

has cartilage which is easily infected (less blood circulation)

nursing management for nutrition

high in fats and carbs

therapy for carbon monoxide poisoning

hyperbaric oxygen therapy

Complications within the GI system during the acute phase

ileus -why it is important to wait to feed the patient after the bowel sounds have returned

Escharotomy

incision that releases scar tissue that prevents the body from being able to expand, enables chest excursion in circumfrential chest bums.

what is important to quickly do for a patient with burns to the face or chest?

intubate quickly before the fluid shift occurs and the airway is restricted

how do should a patient be positioned when suffering from burns to the head or face?

keep the HOB elevated >30 degrees

what is the ice burg effect related to electrical burns?

outside looks fine, inside is lethal --> more damage to the inside of the body than the outside (the outside may even look normal)(the internal damage is what is causing all the major problems)

Complication within the musculoskeletal system during the acute phase

scar tissue begins to form -requires gentle ROM

complications during the rehabilitation phase?

scaring & contractures *splinting and positioning is very important*

white area around a burn means?

that it is thicker and deeper (most likely a third degree burn)

what happens to a persons hematocrit levels with a burn?

the hematocrit increases (concentrated) as a result of plasma loss until fluid resuscitation, then hemodiluted occurs

signs of carbon monoxide poisoning

the skin color turns bright red - *"cherry red skin"*

full thickness burns

third and fourth degree

fluid resuscitation based on urinary output

titrate to yield at least 30 mL urine/ hr

what is the goal of escharotomy

to improve circulation

the extent of burns is determined by?

total body surface area (TBSA)

ROM for burn care?

want to promote in order to prevent contractures -do not want to do too soon after graft is placed

location- hands

worry about ADLs


Related study sets